MINISTRY OF PUBLIC HEALTH OF UKRAINE

Department of human resources policy, education and science

Testing Board

Student ID Last name

Variant ______

Test items for licensing examination

Krok 2 STOMATOLOGY General Instruction Every one of these numbered questions or unfinished statements in this chapter corresponds to answers or statements endings. Choose the answer (finished statements) that fits best and fill in the circle with the corresponding Latin letter on the answer sheet.

ББК 54.1я73 УДК 61 Authors of items: Aksonova Ye.A., Amosova L.I., Andrianova I.I., Artiukh V.M., Avdieiev O.V., Babenko L.M., Bas O.A., Bedeniuk O.A., Beliaieva L.H., Bielikov O.B., Bik Ya.H., Bilous I.V., Bobkova S.A., Bosa L.F., Chernov D.V., Chyhrynets V.N., Chyzhevsky I.V., Denysova O.H., Derkach L.Z., Dmytrenko R.R., Dontsova D.O., Dubrovina O.V., Dubrovina-Parus T.A., Duda K.M., Eismund A.P., Fastovets O.O., Flis P.S., Furdychko A.I., Haiduk R.V., Hanchev K.S., Herasym L.M., Hirchak H.V., Holik V.P., Holovko N.V., Holubieva I.M., Honcharenko V.A., Honcharuk L.V., Hordiychuk M.O., Horlenko O.V., Hrad A.O., Hrekuliak V.V., Hrynovets V.S., Hubanova O.I., Hurzhiy O.V., Ivanitska O.V., Ivchenko N.A., Kalinichenko Yu.A., Kaskova L.F., Katurova H.F., Kharchenko O.I., Kharkov L.V., Kobryn O.P., Kolesnyk V.M., Kolomeichuk B.Ya., Konovalov M.F., Kosarieva L.I., Koval O.V., Kovalchuk L.P., Kril A.Y., Krupnyk N.M., Kryzhanivska O.O., Kuchyrka L.I., Kutelmakh O.I., Kyrychenko V.M., Kyryliuk L.M., Kyryliuk M.I., Larionov I.M., Larionova L.V., Lokes K.P., Lunhu V.I., Lysenko Yu.H., Lysiuk S.V., Malakhovska A.O., Marchenko N.V., Mazurina I.O., Mikhalova A.O., Miziuk L.V., Moiseitseva L.O., Morozova M.M., Muzychina H.A., Nemish T.Yu., Nesyn O.F., Odzhubeiska O.D., Onyshchenko S.I., Orlovsky V.O., Ostafiychuk M.O., Ozhohan Z.R., Palis S.Yu., Pantus A.V., Parasochkina V.V., Pasechnyk A.M., Pidlubna L.S., Piuryk V.P., Posolenyk L.Ya., Prodanchuk A.I., Prots H.B., Raida A.I., Riaboshapko O.A., Riznyk S.S., Romanenko I.H., Rozumenko O.P., Ruzin H.P., Samsonov O.V., Senchenko O.M., Shcherbyna I.M., Shmat S.M., Shubladze H.K., Shuvalov S.M., Shuturminsky V.H., Siry O.M., Stavytsky S.O., Strelchenia T.M., Svirchkov V.N., Sydorchuk S.H., Sydorenko I.V., Sydorova A.I., Sylenko Yu.I., Tereshyna Z.L., Tiuhashkina Ye.H., Tkachenko P.I., Tril S.I., Tsentilo V.H., Udod O.O., Valchuk O.H., Yatsenko I.V., Yermakova I.D., Yeroshenko A.V., Yevtushenko L.H., Yudina O.O., Zavoiko L.M. and Committees of professional expertise.

Item reviewers. Bezvushko E.V., Bulbyk O.I., Chyzhevsky I.V., Dmytriieva A.A., Fastovets O.O., Gerelyuk V.I., Ilenko N.M., Kaskova L.F., Lungu V.I., Muntian L.M., Novikov V.M., Ostapko O.I., Smagliuk L.V., Solovey S.I., Tril S.I., Tsentylo V.G., Volynets V.M., Volyak M.N.

The book includes test items for use at licensing integrated examination “Krok 2. Stomatology” and further use in teaching. The book has been developed for students of stomatological faculties and academic staff of higher medical educational establishments.

Approved by Ministry of Public Health of Ukraine as examination and teaching publication based on expert conclusions (Orders of MPH of Ukraine of 14.08.1998 №251, of 27.12.1999 №303, of 16.10.2002 №374, of 29.05.2003 №233).

© Copyright Testing Board. Krok 2 Stomatology (англомовний варiант, iноземнi студенти) 2017 рiк 1

1. A 27-year-old man complains of teeth filled. Gradually the tooth assumed grayish mobility in his upper and lower jaws. color. Objectively the 11 is filled, discolored, Objectively: dentition is intact. Central stable, painless on percussion. Deep occlusi- occlusion is determined. What examinati- on is observed. What part of the clinical on methods should be applied? presentation contraindicates installation of an all-porcelain crown? A. Analysis of diagnostic models of the jaws B. Gnathodynamometry A. Deep occlusion C. Masticatiography B. Front teeth defects that cannot be D. X-ray corrected with fillings E. Electromyography C. with tooth deformati- on and discoloration 2. A 47-year-old man complains of mobility D. of the artificial crown on the 36 tooth, whi- E. Devitalized teeth defects that cannot be ch was made 2 years ago. Objectively: the corrected with dental inlays 36 tooth is covered with full metal swaged crown. Crown decementation and 6. A 18-year-old woman needs a denture. demineralization are observed. What is the Objectively: the 21 tooth is dark gray in cause of such complication? color, devitalized; orthognathic occlusion is observed. The tooth is to be covered with A. The crown is loose at the tooth cervix plastic crown. What plastic should be used B. The crown edge is embedded into the to make the crown? gingival pocket C. There are interdental contacts A. Sinma-M D. The crown contacts with antagonistic B. Phtorax teeth C. Protacryl-M E. Useful life of the crown is exceeded D. Bacryl E. Etacryl 3. A 48-year-old man complains of itchi- ng . Objectively: the gums are firm; 7. A 58-year-old patient has made an appoi- cold water causes short-time pain in the ntment to make a denture. Objectively: the teeth. X-ray imaging shows osteosclerosis 22 tooth is firm and intact. The alveolar crest of the alveolar septum: close-meshed bone is atrophied; the is flat. Removable structure, the alveolar septum height and denture is to be made. What approach intactness of the cortical plates is retained. regarding the 22 tooth should be chosen by What is the most likely diagnosis? adentist? A. Initial periodontosis A. To make a telescopic crown B. Atrophic B. To retain the tooth C. Initial periodontitis C. Removal of tooth pulp D. Periodontitis, stage I D. Tooth extraction E. Periodontosis, stage I E. To make a stump crown 4. A 60-year-old man complains of stabbi- 8. A patient needs his 26 tooth extracted. ng pain near the root of the on the After application of tuberal anaesthesia right, which develops during eating, especi- he developed general fatigue, nausea, and, ally sour food. Objectively: there is a swelli- later, severe itching and skin rashes. What ng in the right submandibular area. On complication occurred in the patient? palpation the is dense and enlarged. Excretory duct orifice of the A. Urticaria right submandibular gland is dilated and B. Anaphylactic shock produces mucopurulent secretion. What is C. Collapse the most likely diagnosis? D. Quincke’s edema E. Unconsciousness A. of the submandibular gland B. Calculous of the sublingual 9. For a 24-year-old woman a Richmond gland crown is being made to restore the crown C. Acute suppurative lymphadenitis of the central maxillar incisor. The cap is D. Acute completed. What is the next step? E. Adenophlegmon of the right submandi- bular area 5. A 19-year-old woman, an actress, complai- ns of discoloration of her left maxillary central incisor. One year ago the pulp of this tooth was removed and the tooth was Krok 2 Stomatology (англомовний варiант, iноземнi студенти) 2017 рiк 2

A. To fit the cap on the tooth stump and fever up to 38, 6oC. Objectively: there is a place the post in the root canal soft tissue edema in the right parotid regi- B. To solder the post with the cap on, the skin is slightly strained, without di- C. To fit the cap and the post to the tooth scoloration. There is a dense painful infi- root ltration 2,5x3,5 cm large, the skin over it D. To make the combination dental crown exhibits limited mobility. The mouth can be E. To fixate the tooth with cement fully opened, the around the orifice of the salivary duct is unchanged, 10. A 25-year-old patient complains of saliva is transparent. What is the most likely profuse gingival hemorrhages, pain in the diagnosis? oral cavity, weakness, fatigue, fever up to 38oC. These presentations appeared a A. Acute lymphadenitis week ago. Objectively: the patient is pale, B. Exacerbation of chronic adynamic. Examination of the C. Abscess of the parotid-masseteric region reveals multiple hemorrhages, friable gums, D. Acute non-epidemic parotitis necrotic areas on the tops of gingival papi- E. Epidemic parotitis llae, as well as enlarged, soft, painless lymph nodes. The oral mucosal lesion can be a 14. A 33-year-old woman has been admi- symptom of the following disease: tted to the dentofacial department with complaints of pain and edema in the right A. Acute leukemia submandibular region, body temperature B. Chronic leukemia rise up to 39, 5oC. Objectively: the patient C. Vincent’s has asymmetric face because of soft tissue D. Hypovitaminosis C edema of the right submandibular region, E. Intoxication with heavy metal salts palpation reveals a dense infiltration, the skin over it is hyperemic, cannot make a 11. A 67-year-old patient complains of fold. The 46 tooth has a deep carious cavity. recurrent erosion on the red border of What is the most likely diagnosis? the lower . Objectively: the erosion is oval in shape, 0,8х1,3 sm in size, covered A. Submandibular phlegmon on the right in thin scabs that reveal glossy surface B. Acute submandibular sialadenitis with punctate bleeding, when removed. C. Acute suppurative periostitis of the There are atrophic areas of the red border detected. Infiltration elements are absent. D. Acute suppurative submandibular The submandibular lymph nodes are not lymphadenitis enlarged. What is the provisional diagnosis? E. Acute right-sided osteomyelitis of the mandible A. Manganotti’s abrasive precancerous chei- litis 15. Having recovered from acute respiratory B. , erosive ulcerative form disease, a patient has made an appointment C. Keratoacanthosis with a dentist. The patient complains of D. Bowen’s disease pain in the gums, foul breath, inability to E. glandularis take food, general weakness. Objectively: the gums are hyperemic, swollen, covered 12. A 53-year-old patient complains of pain with necrotic dirty-gray coating; the gums and clicking in the left temporomandibular bleed when the coating is removed. Microbi- joint. Objectively: the face is symmetrical, ological study of tissues revealed a great palpation of the lateral pterygoid muscles is number of cocci, bacilli, fusobacteria, and painful on the left side. Mouth opening is spirochaete. Specify the drug for etiotropic reduced. Tomography shows the bone outli- treatment: ne of joint surfaces to be smooth. Which di- sease of those listed below corresponds with A. Metronidazole this clinical presentation? B. Galascorbinum C. Potassium permanganate A. disfunction D. Tripsin B. Rheumatic arthritis E. Carotolinum (Betacarotene) C. Deforming arthrosis D. Acute posttraumatic arthritis 16. A 35-year-old patient complains of itch, E. Joint ankylosis burning and edema of . These presentati- ons occured a week ago. Objectively: there is 13. A 22-year-old patient complains of a pai- reddening of the red border and skin, especi- nful swelling in the right . A ally in the area of the mouth corners, there week earlier the patient received a are also vesicles, scabs, small cracks against that healed under the purulent the background of erythematous affection crust. Over the past two days the patient of the red border. What is the most likely had been observing progressing pain and diagnosis? Krok 2 Stomatology (англомовний варiант, iноземнi студенти) 2017 рiк 3

A. Deep A. Acute eczematous cheilitis B. Open bite B. Multiform exudative erythema C. False C. Acute herpetic cheilitis D. True prognathism D. Allergic contact cheilitis E. Cross bite E. Exudative form of exfoliative cheilitis 21. A 43-year-old woman complains of 17. A 47-year-old patient presents with mobility and displacement of her upper rounded bone protrusions 0,7-0,8 cm in si- front teeth. Objectively: dental formula is ze on the inner surface of the edentulous 17 16 15 14 13 12 11 21 22 23 24 25 26 27 mandible in the premolar area. The denture 47 46 45 44 43 42 41 31 32 33 34 35 36 37 . for this patient should have: Teeth 12 11 21 22 are slanted towards the vestibular side, diastema and tremata A. Elastic liner are observed, I-II degree teeth mobility is B. Kemeny clasps detected. Select the orthodontic appliance C. Metal base for correction of teeth misalignment as a D. Orifices for the exostoses part of complex treatment of periodontal di- E. Dentogingival clasps sease: 18. A 78-year-old patient is completely A. Palatal plate with vestibular arch edentulous. He has been wearing dentures B. Bynin appliance for 19 years. The patient complains of poor C. Schwartz appliance fixation of the upper denture. Objectively: D. Katz crown the lower third of face is shortened, the E. Palatal plate with inclined plane alveolar processes of both jaws are markedly atrophied, the palate is flat. Mucous 22. A 44-year-old patient consulted a dental membrane in the denture-supporting area surgeon about constant acute pain in the is atrophied. How often should the dentures upper jaw region on the left that aggravates be remodelled or restored? during teeth joining. The pain appeared 3 days ago. Objectively: the face is symmetric, A. Every 3-4 years mouth opening is not limited. The crown B. Every 6 months of the 26 tooth is half-decayed. Probing C. Once a year of the carious cavity is painless. Percussi- D. Every 7 years on of the 26 tooth provokes acute pain. E. Every 10-12 years Mucous membrane of the alveolar process 19. A 12-year-old boy complains of pai- is edematic, hyperemic at the level of the 26 nful and bleeding gums on his upper jaw. tooth. The 26 tooth had been treated before. Objectively the in the area What is your provisional diagnosis? of the 13, 12, 11, 21, 22, 23 teeth is swollen, A. Exacerbation of of hyperemic, deformed due to overgrowths. the26tooth Gingival papilla cover the crowns by 1/3 of B. Acute suppurative periodontitis of the 26 their height, bleed on touch. Upper front tooth teeth are overcrowded. X-ray shows no C. Acute of the 26 tooth pathological changes of the . D. Acute suppurative periostitis of the left What drugs should be administered for topi- upper jaw extending from the 26 tooth cal treatment in the first place? E. Periodontitis of the 26, 27,and 28 teeth A. Nonsteroidal antiinflammatory drugs 23. When a prosthodontist was preparing B. Sclerosants the patient’s tooth, the patient had epileptic C. Steroidal antiinflammatory drugs seizure that was subsequently terminated. D. Keratoplastic agents What mistake had been made by the doctor? E. Cytostatic agents A. No inquire into the patient anamnesis 20. A girl is 8 years old. She complains B. No inquire into the antecedent anamnesis of impaired mastication. Objectively: on C. No anaesthesia examination of the oral cavity the cutting D. Crude preparation edges of her lower incisors touch the palati- E. Did not decline the appointment ne mucosa in the frontal area; the upper frontal teeth overlap with the lower ones 24. Parents of a 3-year-old child report that by full height of their crowns. On the lower the child suffers from constant pain in the jaw the occlusal curve of the front teeth is upper front teeth. Objectively: the coronal markedly concave. Make the provisional di- part of the 61 tooth is gray and decayed. agnosis: Probing of the root canal orifice is painful and accompanied by bleeding. The tooth percussion provokes acute pain. Mucosa is Krok 2 Stomatology (англомовний варiант, iноземнi студенти) 2017 рiк 4 hyperemic, edematic and painful. Palpation cles and erosions covered in grayish fibri- in the region of the 61 and 62 teeth reveals a nous coating on the hyperemic and swollen fistula. What is your provisional diagnosis? labial and buccal mucosa. Nikolsky’s sign is negative. What is the most likely diagnosis? A. Exacerbation of chronic periodontitis B. Acute suppurative periodontitis A. C. Acute diffuse pulpitis B. vulgaris D. Chronic granulating periodontitis C. Acute herpetic stomatitis E. Exacerbation of chronic pulpitis D. Nonacantholytic pemphigus E. Dermatitis herpetiformis (Duhring’s 25. A 30-year-old patient complains of disease) a toothache caused by hot and cold sti- muli. The pain irradiates to the ear and 29. A 56-year-old man complains of temple. Previously this tooth presented with enlarged lower lip, pain induced by hot, spontaneous nocturnal toothache. Objecti- sour, salty, and bitter foods, and lips glui- vely: on the occlusal surface of the 37 tooth ng together in the morning. The lower lip there is a deep carious cavity communicating has been gradually enlarging and developi- at one point with the tooth cavity. Probing ng lumps his whole life. On examination: at the communication point, as well as cold the lower lip is enlarged. The middle thi- stimulus, causes acute pain. The pain persi- rd of the Klein’s zone presents with several sts for a long time. Electric pulp test result small red dots with openings that discharge is 5 microamperes. What is the most likely clear drops. The red border is dry and peeli- diagnosis? ng, there are fissures and erosions. What is the most likely diagnosis? A. Exacerbation of chronic pulpitis B. Acute diffuse pulpitis A. Cheilitis glandularis C. Exacerbation of chronic periodontitis B. Cheilitis exfoliativa D. Chronic concrementous pulpitis C. Cheilitis actinica E. Acute suppurative pulpitis D. Allergic contact cheilitis E. Atopic cheilitis 26. A 52-year-old woman complains of peri- odical appearance of a gingival fistula in the 30. A 57-year-old retired man complains area of the 15 tooth. The tooth had been of attacks of burning pain and rashes on treated 1,5 years ago for caries. Objecti- the skin of his face and oral mucosa on vely: the 15 tooth is filled. In the root apex the right. Anamnesis: a course of radiati- projection there is a fistula; purulent exudate on therapy for treatment of the gastric di- discharges on pressure. Tooth percussion is sease, past case of chickenpox. Objecti- painless. On X-ray: the root canal is not fi- vely: along the third branch of the trigemi- lled, there is a destruction focus with blurred nal nerve the skin of the face presents wi- margins near the root. Make the diagnosis: th isolated erosions covered in fibrinous coating. There are multiple vesicles on the A. Chronic granulating periodontitis hyperemic and swollen oral mucosa. Right- B. Exacerbation of chronic granulating sided lymphadenitis is observed. What di- periodontitis agnosis is the most likely? C. Chronic fibrous periodontitis D. A. Herpes zoster E. Chronic granulomatous periodontitis B. Neuralgia C. Murrain 27. A man was diagnosed with D. Acute recurrent herpes abscess. What approach should be chosen E. Neuritis for abscess dissection? 31. A 42-year-old woman complains of acute A. Triangular dissection of the hard palate pain in her lower jaw, teeth mobility, high area fever. The condition persists for 2 days. On B. Linear dissection parallel to the hard clinical examination a doctor diagnosed her palate raphe with acute mandibular osteomyelitis. What C. Linear dissection perpendicular to the tactics regarding the mobile teeth should the hard palate raphe doctor choose? D. Pus aspiration with a syringe E. Abscess puncture 28. A 32-year-old patient presents with body temperature of 38, 9oC, general fati- gue, impaired speech, inability to eat. This condition has been recurring for the last 4 years in autumn and spring. There are vesi- Krok 2 Stomatology (англомовний варiант, iноземнi студенти) 2017 рiк 5

A. Extraction of the causative tooth only A. Prognathism, distal mandibular di- B. Extraction of all mobile teeth splacement C. Conservative treatment of the causative B. Prognathism, maxillary macrognathia tooth C. Retrusion of the lower jaw frontal area D. Conservative treatment of all mobile teeth D. Progenia, mandibular macrognathia E. Extraction of the causative tooth, E. Protrusion of the upper jaw frontal area trepanation and treatment of all mobile teeth 36. A 10-month-old child is fussy, refuses to eat. Disease onset was 2 days ago. The 32. A 7-year-old boy complains of increased child is been treated by a pediatrician for body temperature, up to 38oC, headache, pneumonia, receives antibiotics and sulfani- sore throat. Objectively: there are erosions lamides. Objectively: the oral mucosa is on the slightly hyperemic mucosa of the hyperemic, swollen; there is whitish coati- , anterior palatal bars, and tonsils. ng on the mucosa of the , lips, soft The submandibular lymph nodes are slightly and hard palate; coating removal can cause enlarged, painless. Name the causative agent erosions. Submandibular lymph nodes are of this disease: enlarged. What is the most likely diagnosis? A. Coxsackie virus A. Acute candidal stomatitis B. Herpes simplex virus B. Acute herpetic stomatitis C. Epstein-Barr virus C. D. Klebs-Loeffler bacillus D. Allergic contact stomatitis E. Bordet-Gengou bacillus E. Chronic candidal stomatitis 33. A 6-year-old boy complains of a cavi- 37. A 20-year-old man complains of sharp ty in the prevoiusly treated tooth. Objecti- pain in the mouth, increase of body vely: in the 85 tooth there is a carious cavity temeperature up to 38, 5oC, headache and within mantle dentin; the dentin is dense aching joints, general weakness. The disease and pigmented; probing of the cavilty floor onset was 3 days ago due to overexposure to and walls, thermal stimuli, and percussi- cold. Objectively: the red border is covered on are painless. Sensitivity of the dentin- with hemorrhagic scabs, oral mucosa has enamel junction is observed during the cavi- large erosions and ulcers merging with ty preparation. Make the diagnosis: each other and covered with grayish-white coating against the background of diffuse A. Chronic median caries hyperemia. Conjunctivitis is observed. The B. Chronic deep caries skin of the forearms has erythematous spots C. Acute median caries 1,5 cm in diameter, with blisters in their D. Acute deep caries center. What is the most likely diagnosis? E. Chronic superficial caries A. Stevens-Johnson syndrome 34. A patient consulted a dentist about a B. Erythema multiforme cosmetic defect in the cervical region of C. Drug-induced stomatitis the upper and lower canines. Various sti- D. Lyell’s syndrome muli cause no pain. Objectively: there are E. Behcet’s syndrome V-shaped defects on the vestibular surface in the cervical area of the upper and lower 38. Parents of a 12-year-old child are canines. Their surface is smooth, glossy, concerned about the child having white hard. There is no reaction to probing and spots on the frontal teeth of the upper jaw; cold stimuli. What treatment should be the spots appeared half a year ago. Objecti- administered? vely: there are chalky spots detected in the cervical zone vestibular surfaces of the 13, A. Filling of the defects 12, 11, 21, 22, and 23 teeth. The enamel in B. Metal crowns those spots is dull; probing revealed it to C. Applications with 10% solution of calcium be pliant and coarse. The anamnesis states gluconate short-time pain caused by chemical stimuli. D. Application of fluorine lacquer What is the provisional diagnosis? E. Medical intervention is unnecessary A. Acute initial caries 35. A boy is 10 years old. He complains B. Chronic initial caries of sloped chin and impaired mastication. C. Acute superficial caries Anamnesis states formula feeding. Objecti- D. Systemic hypoplasia of enamel vely: corellation of the 6th teeth is of the E. Dental fluorosis Angle’s II class. Sagittal fissure is 7 mm. Eschler-Bittner test is positive. What is the 39. A 22-year-old woman complains of pain most likely diagnosis? caused by hot food and bursting sensation in the tooth. Half a year ago she presented Krok 2 Stomatology (англомовний варiант, iноземнi студенти) 2017 рiк 6 with brief bouts of pain in the night, whi- The rest of her teeth present with the I-II ch over time increased in duration. Objecti- degree of mobility. Generalized periodonti- vely: there is a large carious cavity in the tis is observed. What denture construction 24 tooth, which opens to the dental cavity, would be optimal in this case? deep probing is painful. Electric pulp test is 80 microamperes. What is the most likely A. Removable dental splint diagnosis? B. Clasp-retained (bugel) removable partial denture A. Chronic gangrenous pulpitis C. Removable laminar denture B. Chronic concrementous pulpitis D. Fixed dental bridge C. Chronic hypertrophic pulpitis E. Metal-based denture D. Acute suppurative pulpitis E. Chronic fibrous pulpitis 44. A 55-year-old patient requires a denture. Objectively: Kennedy’s I class dentition 40. A 14-year-old girl complains of bleedi- defect; the 16, 17, 18, 26, 27, and 28 teeth ng gums and foul smell from her mouth. are missing. The patient presents with fi- Objectively: gingival mucosa is hyperemic, xed occlusion. The 15 and 25 teeth have pastose, hemorrhaging. Schiller-Pisarev test low crowns with poor anatomic contours, is positive. Papillary marginal alveolar index intact. Clasp-retained (bugel) removable is 70%. Fedorov-Volodkina Hygiene Index partial denture is being made for the pati- equals 3. X-ray of the frontal area of jaws ent. What fixation system would be optimal demonstrates retained cortical plate. Make in this case? the diagnosis: A. Telescopic fixation A. Chronic generalized catarrhal gingivitis B. Attachments B. Chronic generalized periodontitis C. Roach clasp (clammer) C. Acute generalized catarrhal gingivitis D. Aker-Roach combined clasp (clammer) D. Chronic generalized hypertrophic gingivi- E. Continuous clasp (clammer) tis E. Exacerbation of chronic generalized 45. A patient needs a clasp-retained (bugel) periodontitis removable partial denture. It is planned to study the jaw model by means of a 41. A 40-year-old man had his root canal parallelometer in order to determine the of the 34 tooth filled due to chronic fi- required depth of the undercuts on the brous periodontitis. Soon the treated place abutment teeth. Specify the length of the became painful. On X-ray the root canal of measuring rods used for this purpose: the34toothisfilled to the root apex. What tactics should the dentist choose to manage A. 0,25 0,50 0,75 the pain? B. 0,15 0,40 0,65 C. 0,20 0,45 0,70 A. Toprescribe physiotherapeutic procedures D. 0,30 0,55 0,80 B. To rinse with antiseptic mouthwash E. 0,35 0,60 0,85 C. To make insicion along the mucogingival fold 46. In a prostodontic clinic a partial lami- D. To provide conduction anesthesia nar denture for the upper jaw is being made E. To provide infiltration anesthesia for a 53-year-old patient. Objectively: dental formula is 14, 13, 12, 11, 21, 22, 23, 24, 27. 42. A 38-year-old patient with chronic The teeth are firm, clinical crowns are tall generalized periodontitis has been referred with pronounced equator. X-ray shows no for orthopedic treatment. Objectively: periapical changes in the periodontium of dentitions are without gaps, the 12, 11, 21, the abutment teeth. What clasp fixation is and 22 teeth are pulpless and exhibit I grade optimal for this patient? mobility. The other teeth are firm. What is the most aesthetic dental splint for the A. Planar anterior teeth? B. Sagittal C. Diagonal A. Mamlok’s splint D. Transversal B. Ring splint E. Point C. Soldered combined crowns D. Cap splint 47. A 20-year-old man complains of missi- E. Mouthguard ng tooth on the upper right jaw, aesthetic defect. Objectively: the 12 tooth is absent, 43. The 40-year-old woman complains of adjacent teeth are intact, firm, with disti- inability to properly masticate due to the nct anatomical shape and tall crowns. Direct loss of the following lateral teeth: 18, 16, occlusion is observed. During the interview 15, 25, 26, 28, 38, 35, 36, 44-46, and 48. the patient was found out to have congeni- Krok 2 Stomatology (англомовний варiант, iноземнi студенти) 2017 рiк 7 tal heart disease. What denture construction region. The poorly circumscribed, painful would be optimal in this case? formation infiltrates the surrounding tissues. At the right side of neck in front and behi- A. Adgesive dental bridge nd the sternocleidomastoid muscle there are B. Plastic dental bridge enlarged, dense, mobile lymph nodes. The C. Porcelain-fused-to-metal dental bridge right naso-buccal groove is flattened, the with 14 and 13 abutment teeth corner of the mouth is downturned. The D. Plastic-fused-to-metal dental bridge mouth opens freely. The are pronounced E. Swaged-soldered dental bridge symptoms of the right facial nerve paresis. What disease can be suspected? 48. The 15 tooth must be extracted. The tooth crown is retained. What instrument A. of the parotid salivary should be used in this case? gland B. Chronic parotitis A. Forceps with S-shaped handles C. Actinomycosis of the parotid-masseteric B. Straight forceps region C. Bayonet forceps D. Chronic lymphadenitis D. Left-sided forceps with S-shaped handles E. of the parotid E. Right-sided forceps with S-shaped handles gland 49. A 7-year-old child complains of pain and 52. A 57-year-old woman came to a denti- swelling in the left submandibular region. st for extraction of the 34 tooth due to The swelling in this region developed 2 days exacerbation of chronic periodontitis. What ago. Objectively: the child is in a satisfactory o instrument would be optimal for tooth condition, body temperature is of 37, 3 C. extraction in the given case? Face is asymmetrical due to the soft tissue swelling in the left submandibular region. A. Beak-shaped non-crushing forceps Palpation reveals a round formation 2x2 cm B. Beak-shaped crushing forceps in size. The formation is mobile, painful, C. Beak-shaped curved forceps unattached to the skin. The 74 tooth is di- D. Straight elevator scolored, percussion is painful. What is the E. Curved elevators provisional diagnosis? 53. A 35-year-old patient has been di- A. Acute serous odontogenic lymphadenitis agnosed with chronic median caries of the of the left submandibular region 36 tooth. There is a Black’s class II cavity B. Acute serous nonodontogenic affecting masticatory surface. What material lymphadenitis of the left submandibular should be chosen for the tooth filling? region C. Acute suppurative odontogenic A. Light-cure microhybrid composite lymphadenitis of the left submandibular B. Glass ionomer cement region C. Silicophosphate cement D. Phlegmonous adenitis of the right D. Light-cure fluid composite submandibular region E. Light-cure microfilled composite E. Lateral cervical cyst 54. A cast clasp-retained (bugel) removable 50. A 32-year-old woman complains of partial denture is being made for a 58-year- tumor-like growth in the mucosa of her left old patient. Impressions are made, centric cheek. Locally: buccal mucosa is of normal jaw relation is determined, plaster casts are color. In the distal area there is a rounded obtained. What is the next stage? elongated growth, soft and elastic, attached to a pedicle sized 0,5х1,5 cm. Make the A. Examination of the working model with a provisional diagnosis: parallelometer B. Transfer of denture frame pattern to the A. Papilloma working model B. Lipoma C. Wax modelling of the denture frame C. Hemangioma D. Duplication of the working model D. Pleomorphic adenoma E. Marking the border seal E. Fibroma 55. A 13-year-old boy complains of general 51. A 52-year-old patient complains of pain weakness, high body temperature up to and swelling in the right parotid region. 39oC, lack of appetite, constant pain in the These manifestations have been present body of the lower jaw. Objectively: observed for about 2 years. Over the last month the is significant asymmetry of the face caused swelling has enlarged, pain has intensifi- by soft tissues swelling in the left buccal ed. Objectively: the face is asymmetric due and submandibular areas. Mouth opening to the dense infiltrate in the right parotid is reduced. Intraoral examination revealed Krok 2 Stomatology (англомовний варiант, iноземнi студенти) 2017 рiк 8 the following: the 34, 35, 36, and 37 teeth Cytological analysis revealed keratinizing are mobile; teeth percussion is painful. epithelial cells. What is the most likely di- The crown of the 36 tooth is completely agnosis? destroyed. The mucosa of those teeth is hyperemic and painful when palpated. Muff- A. Leukoplakia, erosive form like enlargement of the lower jaw alveolar B. Lichen ruber planus, erosive form process is detected. What is the most likely C. Erythema multiforme diagnosis? D. Secondary syphilis E. erythematosus, erosive form A. Acute mandibular odontogenic osteomyelitis 60. A 28-year-old man complains of pain in B. Acute mandibular hematogenous the infraorbital and parotid region on the osteomyelitis left. On examination: hemorrhage occurs in C. Acute mandibular odontogenic suppurati- the lower eyelid and conjunctiva of the left ve periostitis eye, there are signs of crepitation and step D. Ewing’s sarcoma deformity of the eyesocket lower edge. The E. Abscess of the right submandibular area mouth opens by 1 cm. Make the diagnosis: 56. What denture constructions should be A. Zygomatic bone fracture chosen in the cases of multiple adentia duri- B. Malar arch fracture ng the initial period of occlusion change? C. Left articular process fracture D. Traumatic arthritis of the temporomandi- A. Removable partial denture bular joint B. Dental bridge E. Hematoma of the infraorbital region C. Clasp-retained (bugel) removable partial denture 61. A child is 8 years old. There are complai- D. No denture is necessary nts of congested upper incisors. Objectively: E. Removable complete denture the first molars closure is of Angle’s I class, frontal overbite is orthognathic. The 12 and 57. On objective examination a 59-year-old 22 teeth erupt palatinally with space defici- man with the edentulous mandible presents ency of 2/3 of the tooth crown. The 11 and 21 with bone protrusions and mobile areas of teeth are 10 mm each in cross-section. The the alveolar crest. To ensure proper fixation child has inherited father’s facial type with of the denture and even load distribution prognathism and of the central the following functional impression should incisors. Choose the preventive treatment, be made: considering this hereditary pathology: A. Differentiated A. Hotz serial extraction to reduce the dental B. Complete anatomical arch C. Compression B. Jaw expansion to provide the space for the D. Decompression 12 and 21 teeth E. Combined C. Massage of the 12 and 21 teeth area to stimulate their eruption 58. A 49-year-old woman complains of D. Extraction of the 12 and 21 teeth to reduce cosmetic defect of the 11, 21, and 22 teeth, the dental arch which developed over a year ago. Objecti- E. Filing down of the 11 and 21 approximal vely: on the vestibular surface at the equator surfaces to provide the space for the 12 and of the 11, 21, and 22 teeth there are shallow 22 teeth cup-shaped enamel defects that are dense on probing. Cold water induces no pain. Make 62. A 65-year-old woman complains of a the provisional diagnosis: neoplasm in the area of the nasolabial fold on the left, which appeared one month ago. A. Enamel erosion Objectively: there is a gray neoplasm on B. Cuneiform defect the skin of the nasolabial fold on the left, C. Superficial caries markedly keratotic, 3,0х0,5х0,3 cm in si- D. Hypoplasia ze. Neoplastic base is painless, dense, and E. Fluorosis elastic. What is the most likely pathology that results in such clinical presentation? 59. A 47-year-old patient complains of a burning sensation and pain in the mouth. A. Cutaneous horn of the left nasolabial fold Objectively: on the mucous membrane of B. Common of the left nasolabial fold cheeks along the line of teeth contact and in C. Senile keratosis of the left nasolabial fold the corners of the mouth there are multi- D. of the left nasolabial ple polygonal bright red erosions 1,0-1,5 fold cm in diameter located on the hyperkerati- E. Lupus nized plaque and opaque whitish mucosa. Krok 2 Stomatology (англомовний варiант, iноземнi студенти) 2017 рiк 9

63. A 55-year-old patient consulted a denti- st about a rounded tumor-like formation A. Treatment and prevention about 1 cm in diameter located within the B. Prevention red border of his lower lip. Objectively: the C. Retention tumor-like formation protrudes about 5 mm D. Passive above the red border, is dense and grayish- E. Treatment red. The surface of the formation is covered with thin scales that can hardly be removed. 67. A 40-year-old man, a chemical industry What is the most likely diagnosis? worker, notes the development of sour taste of the mouth, pain response to thermal and A. Verrucous precancer of the red border chemical stimuli. On examination: on the B. Manganotti’s abrasive precancerous chei- vestibular surface and cutting edge of the litis front teeth there are enamel defects wi- C. Precancerous limited hyperkeratosis of th uneven scalloped margins, chalk-like in the red border color. Make the diagnosis: D. Bowen’s disease E. Erythroplasia of Queyrat A. Acidic necrosis of enamel B. Superficial caries 64. A 38-year-old woman complains of C. Enamel hypoplasia (erosive form) burning pain in her lips and angles of her D. Fluorosis (erosive form) mouth, their dryness. Anamnesis states E. Median caries that she has been suffering from diabetes mellitus for the last 8 years. Objecti- 68. A patient with post-resection upper jaw vely: the red border is dry, congestively defect that invades the nasal cavity has come hyperemic, covered in scales of variable si- to a prosthodontic clinic. What denture is ze. In the angles of the mouth there are recommended in the given case? fissures covered in white coating, the skin is macerated. What ointment should be A. Replacement denture with obturating prescribed for topical treatment in the gi- element ven case? B. Floating obturator C. Mouthguard A. Clotrimazol D. Forming denture B. Interferon E. Replacement denture C. Prednisolone D. Lanolin 69. An injured patient complains of reduced E. Erythromycin opening of the mouth, nose bleeding, skin numbness in the infraorbital and lower 65. A 35-year-old woman complains of lips eyelid region. Objectively: there is face enlargement. The first incident occurred one deformation due to the depression of soft year ago, when she developed lip edema that tissues in the left cheekbone region, step abated quickly, but the lips remained slightly deformity in the middle part of the inferi- enlarged. Three days ago after overexposure or margin of the left orbit and in the area to cold her lips enlarged again. Objectively: of the zygomatic alveolar crest. What is the ptosis, upper and lower lips are markedly most likely diagnosis? enlarged, more on the left, soft, elastic, and painless on palpation; no impressions on the A. Zygomatic bone fracture with di- lip surface are left after pressing it with a splacement of the bone fragments finger. The tongue is swollen, with tuberous B. Fracture of the right zygomatic bone surface and folds on its back. What is the without displacement of the bone fragments most likely diagnosis? C. Le Fort I fracture of maxilla D. Le Fort II fracture of maxilla A. Melkersson-Rosenthal syndrome E. Fracture of the malar arch B. Miescher granulomatous cheilitis C. Quincke’s edema 70. A 30-year-old woman came to a dentist D. Achard’s syndrome with complaints of a slightly bleeding ”sore” E. Meige’s trophedema on her lower lip, which is located at its medi- an and bisects the lip into two even parts. On 66. A 5-year-old child has bad habit of sucki- palpation the lip is swollen and slightly pai- ng on his tongue. At the front area there is nful. What diagnosis corresponds with the a small vertical fissure up to 2 mm in size. given clinical presentation? Neutral closure is observed in the lateral areas of the jaws. The child is diagnosed wi- th open traumatic bite of the I degree. A vestibulo-buccal shield was prescribed for treatment. What is the function of the appli- ance in the given case? Krok 2 Stomatology (англомовний варiант, iноземнi студенти) 2017 рiк 10

A. Chronic labial fissure st. There are complaints of their child havi- B. Tappeiner’s leukoplakia ng traumas of oral mucosa. Objectively: C. Erosive-ulcerative leukoplakia decreased height of the lower face, everted D. Lichen ruber planus, erosive-ulcerative lower lip, deep labiomental furrow, milk form occlusion. The upper incisors fully cover the E. Meteorological cheilitis lower ones; cutting surface of the lower inci- sors make contact with the anterior third of 71. A 30-year-old patient needs to have his the palate. Mesiodistal ratio of the canines 26 tooth extracted because of exacerbati- and the first permanent molars is normal. on of chronic periodontitis. Objectively: the Grouping of the upper and lower front teeth crown of the 26 tooth is decayed by 1/3. is dissimilar. Make the diagnosis according What forceps can be used for this tooth to the Kalvelis classification: extraction? A. Deep traumatic overbite A. S-shaped forceps with a projecting tip on B. Deep incisor overbite the left beak C. Deep neutral occlusion B. S-shaped forceps with a projecting tip on D. Deep prognatic (roof-shaped) occlusion the right beak E. - C. Straight forceps D. Straight elevator 76. A 46-year-old patient complains of E. S-shaped forceps without projecting tips mastication disorder caused by the lack of the 34, 35, and 36 teeth. The antecedent 72. A 62-year-old patient came to a dental anamnesis is as follows: the teeth were clinic with complaints of facial swelling, pain extracted 3 months ago due to complication in the lower left jaw, and numb lower lip. On of cariosity. The patient anamnesis: the hi- clinical examination he was diagnosed with story of tonsillitis, rheumatoid arthritis and fracture of the body of mandible on the left, Botkin’s disease. After the appointment wi- edentulous jaws, microstomia. Choose the th this patient the instruments should be optimal construction: sterilized in the following way: A. Limberg’s dental splint A. Specialized procedure B. Weber’s dental splint B. Dry-heat sterilizer C. Guning-Port’s dental splint C. Processing with lysol D. Elbrecht’s dental splint D. Processing with 0,1% chloramine solution E. Vankevych dental splint E. General procedure 73. Prior to dental treatment a 13-year-old 77. Preventive examination of a 5-year-old patient had been administered anaesthesia. child revealed a habit of lower lip biting. The patient complained of itching, tingli- What may develop if the child ng skin of the face, vertigo, nausea, labored keeps this habit? respiration, spontaneous vision impairment. Objectively: pale face, swollen eyelids and A. Anterior bite red border, dilated pupils, thready pulse, B. Prognathic bite and rapid labored respiration with crackles. C. Open bite Make the diagnosis: D. Deep overbite E. Cross-bite A. Anaphylactic shock B. Syncope 78. A 7-year-old child has protruding chin, C. Collapse the lower lip overlaps the upper one. There D. Epileptic attack are diastema and tremata between the lower E. Quincke’s edema incisors, the lower incisors overlap the upper incisors by 2/3 of the crown height. Fi- 74. A 48-year-old patient came to a denti- rst permanent molars demonstrate Angle’s st after the maxillectomy on one side class III relation. Sagittal gap is 3 mm. The conducted 3 days ago. Remaining teeth are correct treatment tactics would be to: firm. Treatment plan foresees making an Oxman’s denture for the patient. What part A. Use Bruckl’s appliance of the denture should be produced first? B. Recommend a complex of myogymnastic exercises A. Fixating C. Use Angle’s slider appliance B. Obturating D. Use Bynin’s appliance C. Resection E. Use Schwartz’s appliance D. Forming E. Substituting 79. A 62-year-old man had been wearing a full removable upper jaw denture. He 75. Parents of an 8-year-old child have complains of inability to use this denture made and appointment with an orthodonti- Krok 2 Stomatology (англомовний варiант, iноземнi студенти) 2017 рiк 11 due to constant burning sensation in the occlusion. X-ray shows a median mandi- hard palate and dryness of the oral cavi- bular fracture. What dental splint would be ty. Objectively: on the hard palate under optimal? the denture base there are an edema and marked hyperemia. What means of A. Flat occlusal splint prevention would be optimal in the given B. Soldered splint on rings case? C. Cap splint D. Weber’s splint A. Shielding of denture base E. Plastic mouthguard B. Strict adherence to polymerization procedure 83. The department of dentofacial surgery C. Careful selection of impression paste admitted a patient who needs repair of a D. Determination of risk group during post-traumatic nose wing defect up to 3,0 cm examination in diameter. The trauma occured six months E. Strict indications for the choice of material ago. What kind of grafting is indicated in this clinical situation? 80. A 42-year-old woman has made an appointment with a prosthodontic A. Grafting with chondrocutaneous flap of office to make a denture. Objecti- the auricle vely: dental formula is as follows: B. Grafting with local tissues of nasolabial or 18....131211 21 22 23 . . . . 28 cheek regions 48 47 46 45 44 43 42 41 31 32 33 34 35 36 37 . C. Grafting with pedicle flap of frontal and The patient has deep occlusion; clinical buccal regions crowns are low; equator is not pronounced. D. Grafting with tubed pedicle flap (Filatov’s The patient suffers from epileptic seizures. flap) What kind of denture should be prescribed E. Free grafting with dermal flap for this patient? 84. A 25-year-old woman consulted a denti- A. Removable partial laminar metal-based st about acute pain in her upper jaw on the denture left. The pain occurs during eating. Objecti- B. Dental bridge vely: on the distal approximal surface of the C. Removable partial laminar plastic denture 26 tooth there is a cavity filled with light with retainers (clammers) soft dentin. Probing causes slight pain along D. Removable partial laminar denture with the dentin-enamel junction, percussion is supporting-retaining clasps (clammers) painless. Cold water causes quickly abati- E. Clasp-retained (bugel) removable partial ng pain. What is the most likely diagnosis? denture A. Acute median caries 81. A 27-year-old woman complains of pai- B. Chronic median caries nful and bleeding gums, with the signs C. Acute deep caries aggravating during eating, and indisposition. D. Chronic fibrous pulpitis One week ago she had a case of URTI. Wi- E. Chronic deep caries thin the last 5 years she periodicaly presents 85. A 27-year-old woman complains of with gingival hemorrhages. Objectively: the recurrent loss of a tooth filling in the lower gums are bright red, markedly swollen; gi- right jaw. Objectively: in the 46 tooth on ngival papillae are friable, bleed on the sli- the masticatory approximal surface there is ghtest touch. There is moderate accumulati- a defect of hard tooth tissues affecting 1/3 on of dental calculus, large amount of of the tooth crown, no tooth discoloration; soft dental deposit. Submaxillary lymph positive, quickly abating reaction to cold sti- nodes are enlarged, painful on palpation. mulus is observed. What denture constructi- On X-ray: osteoporosis of alveolar septa is on would be optimal in this case? observed. What is the most likely diagnosis? A. Dental inlay A. Exacerbation of chronic catarrhal gingivi- B. Combined crown tis C. Porcelain-fused-to-metal crown B. Exacerbation of initial generalized peri- D. Plastic crown odontitis E. Partial crown C. Hypertrophic gingivitis, edematous form D. Hypertrophic gingivitis, fibrous form 86. A 10,5-year-old child complains of pai- E. Acute catarrhal gingivitis nful rash on his lips. Objectively: the red border of the lips is swollen, hyperemic, 82. A 30-year-old man complains of pain in covered in fissures and numerous scabs of his front lower teeth, which he attributes dried blood. The skin of the upper lip has to a trauma to the mental region. Objecti- small blisters containing serous substance, vely: continuous dentition, orthognathic which merge with each other in some places. Krok 2 Stomatology (англомовний варiант, iноземнi студенти) 2017 рiк 12

Maceration and weeping skin also can be A. According to the scheme observed, especially in the corners of the B. The impressions should be desinfected mouth. What is the most likely diagnosis? under a quartz lamp C. The patient should be refused appoi- A. Exematous cheilitis ntment B. Meteorological cheilitis D. No special precautions are required C. Atopic cheilitis E. The orthopaedist should work in gloves D. and a mask E. Exfoliative cheilitis 91. A 3-year-old child received an injury of 87. A 27-year-old man complains of achi- the upper teeth as a result of a fall. Objecti- ng long-lasting pain in the 15 tooth during vely: crowns of the 51 and 61 teeth are eating, especially cold food. Sometimes the embedded deep into the surrounding tissues pain occurs when the temperature changes. with only their cutting edge visible, the gi- Objectively: on the distal surface of the 15 ngival margin is hyperemic, edematic. What tooth there is a cavity filled with softened is the treatment tactics? dentin. Probing is painful. Electroexcitabili- ty of the pulp is 35 microamperes. What is A. Tooth extraction the most likely diagnosis? B. Monitoring C. Reposition A. Chronic fibrous pulpitis D. Endodontic treatment B. Acute deep caries E. Anti-inflammatory therapy C. Chronic deep caries D. Hyperemia of the pulp 92. A 60-year-old patient has been undergoi- E. Exacerbation of chronic pulpitis ng the procedure of checking the complete removable dentures construction and fixing 88. A 56-year-old patient suffering from teeth on wax bases. The following flaws have exacerbation of schizophrenia has been been detected: fissure between the teeth in hospitalised in an oral in-patient department the frontal area and tubercule contact in the with a diagnosis of the lower jaw displaced lateral area. What mistake had been made? fracture in the area of the 34-35 teeth. What method of treatment should be prescribed? A. Anterior occlusion was determined instead of central one A. Osteosynthesis B. Posterior occlusion was determined B. One arch flat occlusal splint instead of central one C. Dual splint C. Lateral occlusion was determined instead D. Weber’s dental splint of central one E. Vankevych dental splint D. Models were plastered in a wrong way in an occluder 89. During preventive examination a 5-year- E. Swabs were crushed, when central occlusi- old child was found to have insufficient on was being determined physiological of tubercles of the deciduous canines. What treatment tactics 93. Dental splint is being designed in a should a doctor choose? prosthodontics clinic for a 39-year-old pati- ent with generalized periodontitis, II degree. A. To shave off the retained canine tubercles Gnathodynamometer is used to measure B. Medical examination once a month until the periodontium load resistance. What the incisors are replaced anatomico-functional data are obtained wi- C. Medical examination every 6 months until th this method? the incisors are replaced D. Medical examination every 6 months until A. Masticatory pressure the canines are replaced B. Masticatory force E. No medical intervention is necessary C. Periodontium pliancy D. Masticatory muscles tone 90. A 25-year-old HIV-infected patient came E. Masticatory efficiency to a clinic of prosthetic dentistry to have a denture made for him. What aseptic and 94. A 45-year-old man complains of impai- antiseptic precautions should be taken? red chewing due to pathologic lower jaw mobility. The patient was diagnosed wi- th false joint in the area of absent 33 and 34 teeth. On X-ray: defect of the mandi- bular body is 0,8 cm in size. The teeth on the fragments on both sides of the defect are intact. What denture would be recommended in this case? Krok 2 Stomatology (англомовний варiант, iноземнi студенти) 2017 рiк 13

A. Oxman’s fixed dental bridge lls, fatigue, fever up to 38oC, muscle pain, B. Tigerstedt’s flat occlusal splint sensations of dryness, burning, and pain in C. Weber’s dental splint the oral cavity, excessive salivation, vesicles D. Entin’s stiff head-chin strap in the interdigital folds, on the lips, oral and E. Tigerstedt’s wire anchor splint nasal mucosa. On examination of the oral cavity there were detected painful bright red 95. A 30-year-old patient complains of pain erosions with polycyclic contours against the and swelling in the area of the left parotid background of inflammation. The following , which occurred 7 days after was observed: scabs on the lips, enlarged he had undergone abdominal cavity surgery. o tongue, impaired speech, salivation up to 4 Objectively: body temperature equals 39 C, liters per day. Make the diagnosis: reduced mouth opening; dry mouth; when the gland is massaged, there is purulent A. Murrain exudate being secreted from its duct. The B. Acute herpetic stomatitis patient can be diagnosed with the following C. Chickenpox disease: D. Measles E. Erythema multiforme A. Acute non-epidemic parotitis B. Acute epidemic parotitis 99. During examination of a 5-year-old child C. Phlegmon of submasseteric space the orthodontist revealed no wear of teeth, D. Parenchymatous parotitis no tremata and diastemata, orthogenic E. Phlegmon of parotid-masseteric region occlusion. Which of the following symptoms detected in the 5-year old child is a sign of 96. A 53-year-old patient complains of an future teeth overcrowding? ulcer on the lateral surface of the tongue. The ulcer appeared 6 months ago in the A. Absence of tremata and diastemata result of a trauma caused by sharp tip of the B. Absence of wear of teeth 37 tooth metal crown. A dentist replaced C. Orthogenic occlusion the crown with the one of better quality D. Orthognathic bite and prescribed keratoplastic drugs. Despite E. Absence of mesial step in the region of these measures the ulcer continues to grow. second temporary molars Lately there has been pain during talking, chewing, and swallowing, with occasional 100. An 8-year-old child has been clini- irradiation to the . Objectively: on cally diagnosed with exacerbation of chronic the lateral surface of the tongue there is periodontitis of the 84 tooth. The crown is a painful ulcer with uneven raised dense decayed by 1/2. What is the optimal tactics margins and lumpy floor covered with grayi- of dental treatment? sh necrotic coating. What is the most likely diagnosis? A. Extraction B. Endodontic treatment A. Cancer of the tongue lateral surface C. Endodontic treatment and drug therapy B. Trophic ulcer D. Drug therapy C. Traumatic ulcer E. Opening along the mucogingival fold, D. Vincent’s necrotizing ulcerative stomatitis drug thrapy E. Tuberculous ulcer 101. A 7-year-old child is diagnosed with 97. A 15-year-old girl complains of chronic granulating periodontitis of the 55 toothache that persists for a day and tooth. Additionally accompanying diagnosis increases on biting. Objectively: in the 36 of rheumatic endocarditis is made. What tooth there is a deep carious cavity non- treatment tactics should a dentist choose? communicating with the dental cavity. No reaction to the thermal stimuli is observed, A. Tooth extraction probing of the carious cavity floor is painless. B. Endodontic treatment Vertical percussion is markedly painful. Gi- C. Endodontic treatment and physical ngival mucosa in the area of the 36 tooth is therapy unaltered. X-ray presents with no alterati- D. Case monitoring ons. Make the diagnosis: E. Endodontic treatment and case monitori- ng A. Acute serous periodontitis B. Acute suppurative pulpitis 102. Parents of an 8-year-old child complain C. Acute suppurative periodontitis of rashes in the child’s oral cavity. Lately the D. Acute serous pulpitis child has been inert, refused to eat. On the E. Exacerbation of chronic periodontitis oral mucosa there are small round erosions with clear margins. There are vesicles with 98. A 35-year-old patient, a veterinarian, turbid content on the child’s face and scalp. came to a dentist with complaints of chi- Make the provisional diagnosis: Krok 2 Stomatology (англомовний варiант, iноземнi студенти) 2017 рiк 14

107. A laminar denture for the lower jaw is A. Chickenpox being made for a 54-year-old patient. Base B. Measles plate wax is used during the laboratory stage C. Hypertensive-hydrocephalic syndrome for wax templates. What group of accessory D. Infectious mononucleosis materials does such wax belong to? E. Scarlet fever A. Modeling 103. A 45-year-old patient is prescri- B. Abrasive bed a dental bridge supported with C. Fixing implants. In the process of preparati- D. Impression on to the prosthodontic treatment there E. Forming were intraosseous screw two-stage implants placed in the area of the 34 and 36 teeth. 108. A man complains of short-term pain How long is the period necessary for implant attacks caused by cold stimuli in the tooth on integration in this case? his upper left jaw, which have been observed for the last 3 weeks. Objective examinati- A. 3 months on of the 25 revealed a Black’s I class cari- B. 2 weeks ous cavity located within mantle dentin. The C. 6 months cavity has narrow opening, its walls and floor D. 10 months are covered with softened dentin. Probing E. 1 year is painful along the dentin-enamel border, percussion is painless, thermometry is pai- 104. A 1-month-old child has problems wi- nful, the pain quickly abates after removal th breastfeeding, insufficiently gains wei- of a stimulus. Electric pulp test is 6 mi- ght. On examination a doctor made the croamperes. Make the diagnosis: diagnosis of abnormal attachment of the tongue frenulum. What method of surgical A. Acute median caries treatment should be chosen in this case? B. Acute deep caries C. Chronic median caries A. Transversal dissection of the frenulum D. Chronic fibrous pulpitis B. Lingual frenulectomy E. Chronic deep caries C. Vestibuloplasty D. Limberg’s relocation of triangular flaps 109. A 54-year-old man complains of tooth E. Dieffenbach’s frenuloplasty hard tissue defects on the upper and lower jaws. Objectively: in the precervical area of 105. A 5-year-old child complains of the upper and lower premolars within the spontaneous pain in an upper jaw tooth on external layer of dentin there are hard ti- the right that aggravates at night and during ssue defects with surfaces meeting under eating cold food. Objectively: the 65 tooth the angle. Surface of the defects is smooth, has a deep cavity communicating with the glossy, and dense. What measure should be tooth cavity. Probing is painful, percussion is taken to prevent further progression of the painless. Cold water causes long-term pain. morbid process in the patient? What is your provisional diagnosis? A. To train him in efficient toothbrushing A. Exacerbation of chronic pulpitis B. To perform functional examination of the B. Acute periodontitis thyroid C. Exacerbation of chronic periodontitis C. To prescribe calcium preparations intake D. Acute serous pulpitis D. To limit sour foods in the diet E. Acute suppurative pulpitis E. To limit sweet foods in the diet 106. A 10-year-old girl complains of sensati- 110. A 35-year-old man has been hospitali- ons of dryness and pain in her lips, whi- zed into a dentofacial unit with complaints ch develop in the summer. On examinati- of mobility of the 38, 37, and 36 teeth and a on: the red border has areas of congestive fistulous tract in the socket of the extracted hyperemia and infiltration, scales and scabs, 35 tooth. The condition has been persisti- that, when removed, result in hemorrhagi- ng for 3 months. Insertion of a grooved ng erosions. The skin surrounding lips is probe into the fistulous tract palpated unaltered. No rashes are detected on the bared coarse bone fragment that easily oral mucosa. What is the most likely di- moved under pressure. X-ray of the lower agnosis? jaw demonstrates a focus of bone tissue A. Actinic cheilitis, dry form destruction, with a spot of dense bone ti- B. Actinic cheilitis, exudative form ssue 0,5х0,3 cm in size. Make the diagnosis: C. Atopic cheilitis D. Allergic contact cheilitis E. Meteorological cheilitis Krok 2 Stomatology (англомовний варiант, iноземнi студенти) 2017 рiк 15

A. Chronic osteomyelitis pain and bleeding of gums in the frontal B. Acute osteomyelitis part of the upper jaw on the left. Two C. Exacerbation of chronic osteomyelitis years ago, the 22 tooth was covered with D. Chronic periostitis a porcelain-fused-to-metal crown. Objecti- E. Actinomycosis vely: between the 21 and 22 tooth is hypertrophied, markedly 111. After extreme overexposure to cold a hyperemic, overlaps the crown of the 22 42-year-old patient complains of headache by 1/3 of its height, bleeds when touched. in the left frontal lobe and the left upper Periodontal pocket between the 21 and jaw. Objectively: the face is symmetrical; 22 tooth is 4 mm deep. Artificial crown is left nasal meatus breathing is obstructed, located on the gingival margin. Radiography and serous-purulent discharge is being reveals resorption of the interalveolar septa produced; palpation of the suborbital area between the 21 and 22 tooth by 1/3 of and further along the mucogingival fold in their height. Specify a priority action in the the 24 and 25 teeth projection reveals sli- treatment of this patient: ght pain. Percussion of these teeth is pai- nless. The 24 tooth is filled. The alveolar A. Removal of the artificial crown process mucosa has no visible alterations. B. Gingivectomy X-ray imaging shows decreased pneumati- C. Anti-inflammatory therapy zation of the left maxillary sinus. What is the D. Curettage of the periodontal pocket provisional diagnosis? E. Sclerotherapy A. Exacerbation of chronic odontogenic 115. A 45-year-old patient came to a maxillary sinusitis prosthodontics clinic. During the objective B. Acute periodontitis of the 24 examination the doctor checked the sagittal C. Exacerbation of chronic periodontitis of movements of the lower jaw. What muscles the 24 are responsible for sagittal movements of D. Acute rhinogenous maxillary sinusitis the lower jaw? E. Acute albuminous periostitis of the left maxilla A. Lateral pterygoid muscles B. Medial pterygoid muscles 112. A 8,5-year-old child is mostly healthy. C. Mandibulohyoid muscle There is a complaint of pain in the upper D. Digastric muscle left tooth, due to it having been physically E. Mentohyoid muscle damaged 3 hours ago. Objectively: 1/2 of the 21 tooth crown is destroyed, the pulp is si- 116. An 18-year-old patient complains of a gnificantly exposed, red, sharply painful and white spot on the vestibular surface of the 21 bleeding when probed. Percussion of the 21 tooth. Objectively: the white spot is located tooth is sharply painful. Choose the optimal near the cutting edge. The spot surface is method of the 21 tooth treatment: glossy, its size remains unaltered on drying. Make the diagnosis: A. Vital amputation B. Vital extirpation A. Local hypoplasia C. Devital amputation B. Fluorosis D. Devital extirpation C. Initial caries E. Biological method D. Enamel necrosis E. 113. A 11-year-old child complains of pain during eating food, especially hot, in the 117. A 20-year-old man complains of lower right lateral tooth. On the masti- spontaneous pain in the 24 tooth, which catory surface of the 46 tooth there is a arose suddenly and persists for about 15 mi- large carious cavity filled with softened nutes. Objectively: the distal surface of the light-brown dentin. The cavity is located wi- 24 tooth exhibits a deep carious cavity with thin parapulpar dentin. In the projection of overhanging walls. The cavity is filled with mesiobuccal pulp horn the carious cavity light softened dentin and communicates wi- communicates with the pulp chamber. Deep th the tooth cavity. The cold stimulus causes probing is painful. Electric pulp test - 60 mi- acute, slowly abating pain. Percussion causes croamperes. Make the diagnosis: no pain response. Select the best method of treatment: A. Chronic gangrenous pulpitis B. Chronic hypertrophic pulpitis A. Vital extirpation C. Acute diffuse pulpitis B. Vital amputation D. Chronic fibrous pulpitis C. Biological method E. Acute focal pulpitis D. Devital amputation E. Devital extirpation 114. A 28-year-old patient complains of Krok 2 Stomatology (англомовний варiант, iноземнi студенти) 2017 рiк 16

118. A 25-year-old patient complains of pain ckened, Dupuytren’s symptom is observed when biting on the 15 tooth. The pain arose on the vestibular surface of the mucogingi- two days ago, has a constant aching nature val fold in the area of the 36 and 37 teeth. and increased significantly over the last day. X-ray of the lower left jaw demonstrates the Objectively: the crown of the 15 tooth is gray, rounded focus of bone tissue destruction wi- the medial contact surface exhibits a deep th clear margins. The roots of the 36 and 37 carious cavity communicating with the tooth teeth are resorbed. Puncture yielded brown cavity. Percussion causes acute pain, the gi- liquid. What is the most likely diagnosis? ngival mucosa in the projection of the 25 tooth root apex is hyperemic. The regional A. Giant cell tumor of the body of mandible lymph node is tender. Radiograph shows an B. Ameloblastoma of the body of mandible ill-defined zone of periapical bone destructi- C. Fibrous dysplasia of bone on. What is the most likely diagnosis? D. Cancer of the body of mandible E. Sarcoma of the body of mandible A. Exacerbation of chronic periodontitis B. Acute serous periodontitis 122. A patient complains of the alveolar C. Acute suppurative periodontitis process deformation on the left upper jaw. D. Chronic granulating periodontitis Objectively: the crown of the 25 tooth is E. Acute serous periodontitis, intoxication destroyed with cariosity. X-ray image of the stage paranasal sinuses shows the left one to have veil-like shading with clear dome-shaped 119. A 27-year-old patient has been margin. X-ray image of the crown of the referred by a prosthodontist for endodontic 25 tooth shows absence of the periodontal treatment of the 45 tooth. Objectively: the fissure at the the palatal root apex. What is 45 tooth crown is destroyed; the lateral the most likely diagnosis? surface of the tongue and the buccal mucosa have patches of grayish macerated epi- A. Radicular cyst that invaded in the maxi- thelium slightly rising above the mucosa llary sinus surface at the points of direct contact wi- B. Chronic rhinogenous maxillary sinusitis th the 45 tooth. The uvula and palatal C. Chronic odontogenic maxillary sinusitis bars are stagnant-red in color; hard palate D. Maxillary sinus mucosal cyst presents with papulae surrounded with red E. Maxillary cancer margin and covered in grayish epithelium. The submandibular, cervical, supraclavi- 123. A patient with ischemic heart disease cular, and subclavicular lymph nodes are has developed ventricular fibrillation. What enlarged and painless. What is the provisi- is the first-priority therapeutic action? onal diagnosis? A. Electric defibrillation A. Secondary syphilis B. Lidocaine injection B. Chronic recurrent C. Adrenaline injection C. , patch stage D. Potassium chloride injection D. Soft leukoplakia (leucoplakia mollis) E. Novocaine amide injection E. Lichen ruber planus 124. A 7-year-old boy came to a dental surgeon with complaints of painful 120. A 43-year-old patient complains of mobility and significant neck exposure of swelling of his right cheek and high body temperature. Objectively the body the lower front teeth. Objectively: the gums oC in the area of the 44, 43, 42, 41, 31, 32, 33, and temperature is 38,2 , facial asymmetry 34 teeth are pale and cyanotic, non-bleeding. caused by the right cheek edema, no skin di- The 42, 41, 31, and 32 teeth exhibit the I-II scoloration, skin can be pinched in a fold, no grade mobility. The overcrowding of the 42, mouth opening reduction. In the oral cavi- 41, 31, and 32 teeth is present. The necks of ty the mucogingival fold in the area of the the 42, 41, 31, and 32 teeth are exposed by 84 and 85 teeth is smoothed out, the fold is 1/2 of the root length, the necks of the 43 hyperemic, fluctuation is observed. The 85 and 33 teeth are exposed by 1/4. What kind tooth is destroyed by cariosity, painless on of denture should be applied in this case? percussion. Make the diagnosis: A. Cast removable splint B. Kurlyandsky’s bar splint C. Cap splint D. Partial crown E. Half-ring splint 121. A 35-year-old woman complains of toothache and thickened body of the mandi- ble. Objectively: the lower left jaw is thi- Krok 2 Stomatology (англомовний варiант, iноземнi студенти) 2017 рiк 17

A. Acute odontogenic suppurative mandi- bular periostitis originating from the 85 tooth A. Catarrhal gingivitis B. Acute odontogenic albuminous mandi- B. Hypertrophic gingivitis bular periostitis originating from the 85 tooth C. Atrophic gingivitis C. Acute odontogenic mandibular D. Periodontitis osteomyelitis originating from the 85 tooth E. Ulcerative gingivitis D. Suppuration of the periapical cyst of the 85 tooth 129. A 22-year-old woman came to a dentist E. Odontogenic abscess of the buccal area for preventive examination. During exami- originating from the 85 tooth nation of the oral cavity the dentist detected a defect of hard tooth tissues in the cervi- 125. A 15-year-old adolescent boy complai- cal area of the 22 tooth within mantle ns of pain in the lower right jaw, which dentin. The dentin is dense and pigmented. increases during chewing, and impaired No reaction to percussion and probing is closure of the teeth. Anamnesis: 2 days ago observed. Make the diagnosis: a trauma was received. Based on objecti- ve examination results and X-ray data the A. Chronic median caries patient was diagnosed with open fracture of B. Cuneiform defect the mandible between the 45 and 46 teeth. C. Acute deep caries Choose the method of treatment: D. Acute median caries E. Necrosis of hard tooth tissues A. Removable braces B. Temporary immobilization 130. Spot X-ray is performed for a 12-year- C. Port’s dental splint old child. The X-ray film shows the 35 tooth D. Rudko’s appliance root to be of nornal length, its walls are E. Ligature parallel to each other and gradually thin out towards the apex. At the entrance the 126. A woman came to a dentist for root canal is slightly smaller in diameter consultation. She is 4 months pregnant. than near the forming apex. The periodontal Objectively: marginal gingiva presents with fissure is uniformly wide along the whole dense gingival papillae, torus-shaped, oval, length of the formed part of the root. In the enlarged up to 1/3 of the tooth crowns. Make apical area of the root it merges with growth the provisional diagnosis: zone. What stages of root and periodonti- um development correspond with this X-ray A. Hypertrophic gingivitis image? B. Catarrhal gingivitis C. Periodontitis A. Unformed apex D. Periodontosis B. Lengthwise growth of a root E. Ulcerative gingivitis C. Open apical foramen D. Unformed periodontium 127. A 53-year-old man complains of E. Fully formed root and periodontium increased teeth sensitivity to chemical sti- muli. Objectively: the gums are pale pi- 131. A 10-year-old boy complains of painful nk, roots are bared by 1/3 of their length. sore in the mouth, which has been persi- Small amount of dental deposit is observed. sting and increasing in size for 1,5 months. The 15, 14, and 24 present with cuneiform Objectively: on the buccal mucosa there is defects. Probing of the bared cervices and a soft shallow ulcer 2 cm in diameter wi- defects is painful. What is the most likely di- th uneven undermined edges. The floor of agnosis? the ulcer is tuberous, covered in yellow- gray coating. The ulcer is surrounded with A. Periodontosis, I degree numerous yellowish tubercles. The regional B. Catarrhal gingivitis lymph nodes are elastic, painful, and matted C. Periodontitis, II degree together. These symptoms are characteristic D. Periodontitis, I degree of the following disease: E. Ulcerative gingivitis A. Tuberculosis 128. A 24-year-old man complains of pai- B. Lichen ruber planus nful and bleeding gums. The condition onset C. Necrotizing ulcerative stomatitis was 3 days ago after the patient had a case D. Cancer of acute respiratory disease. Objectively E. Syphilis the gingival mucosa is swollen, bright-red, bleeds on probing, painful on palpation; ti- 132. A 47-year-old man complains of persi- ps of the gingival papilla are rounded; soft sting dull pain in his upper right jaw. dental deposit is observed. X-ray shows no Anamnesis states that the 13 tooth has been changes in the bone. What is the most likely treated repeatedly to no effect. Objecti- diagnosis? vely: oral mucosa in the area of the 13 Krok 2 Stomatology (англомовний варiант, iноземнi студенти) 2017 рiк 18 and 14 teeth is hyperemic and painless on A. Extracoronal attachments palpation. The crown of the 13 tooth is B. Telescopic crowns destroyed to the gingival margin. Percussi- C. Wire clasps (clammers) on is sharply painful. What conduction D. Cast clasps (clammers) anaesthesia should be applied to extract the E. Intracoronal attachments 13 tooth? 136. A patient diagnosed with rheumatoid A. Infraorbital, incisor, palatal arthritis came to a dental surgeon complai- B. Infraorbital, incisor ning of painful mouth opening characterized C. Infraorbital, palatal, tuberal by pain on both sides of the jaw. The followi- D. Infraorbital, palatal ng is characteristic of rheumatoid arthritis of E. - the temporomandibular joint: 133. A 29-year-old man came to an A. Pain is observed on both sides of the oral surgery department to extract the temporomandibular joint 38 tooth. There are complaints of pain B. Pain is observed on one side of the and reduced mouth opening. Objectively: temporomandibular joint body temperature is 38oC, general condi- C. Pain can be observed on one or both sides tion is satisfactory, slight facial asymmetry of the temporomandibular joint is observed due to soft tissue swelli- D. Clicking is observed in the joint on the ng under the gonial angle on the left. right Inflammatory contracture of the III degree E. Clicking is observed in the joint on the left is observed. In the oral cavity there are edema and hyperemia of mucosa along the 137. A 38-year-old man after a domestic acci- pterygomandibular fold on the left. The dent complains of pain and mobility of his submandibular lymph nodes are enlarged upper teeth, problems with eating. Objecti- and painful on palpation. What conduction vely: soft tissues edema. The 11 and 21 teeth anaesthesia should be applied prior to the are displaced towards the palate, mobile extraction of the 38 tooth? (II degree), painful on percussion. Mucosa surrounding the affected teeth is hyperemic A. Berchet-Dubov anaesthesia and swollen. X-ray demonstrates widened B. Mandibular, lingual periodontal fissure of the 11 and 21. Choose C. Buccal, lingual the treatment method: D. Infiltration E. Tuberal A. Setting of the teeth and their fixation with a flat occlusal splint 134. A 52-year-old man was referred to B. Extraction of the 11 and 21 teeth an in-patient dentofacial department wi- C. Reimplantation of the 11 and 21 teeth th complaints of pain in the submandi- D. Immobilization or mouthguard bular area, aggravating during eating. E. Removal of tooth pulp in the 11 and 21 Anamnesis states frequent exacerbations of teeth inflammatory processes. Main and additi- onal investigations resulted in provisional 138. Parents complain of painfulness and diagnosis of submandibular sialolithiasis. A mobility of the tooth of their 4-year-old chi- concrement 1,5 cm in diameter is localized ld, which developed after the impact with in the body of the gland. What tactics should a wooden object. Objectively: the face is a dental surgeon choose? asymmetrical due to swollen tissues of the upper lip. The 51 tooth is intact, with vesti- A. Gland extirpation bular displacement and the II degree mobili- B. Saliva-producing diet ty, gums around the 51 tooth are hyperemic. C. Physiotherapeutic procedures What provisional diagnosis can be made? D. Puncture biopsy E. Extract the concrement from the gland A. Incomplete dislocation of the 51 tooth and place a blind suture B. Complete dislocation of the 51 tooth C. Contusion of the 51 tooth 135. A 70-year-old patient complains of D. Acute albuminous periostitis impaired mastication. Objectively: the 33 E. Acute suppurative periostitis and 43 teeth on the lower jaw are retai- ned, firm, but have significantly destroyed 139. A 7-year-old child is in a grave condi- crowns. Removable partial denture is requi- tion, teeth of the upper left jaw are pai- red. Choose the optimal method of denture nful, body temperature is 39, 3oC.Thechi- fixation: ld is pale, adynamic; the face is asymmetri- cal due to infiltration in the upper left jaw. The64toothisfilled, painful on percussi- on. The 63 and 65 teeth are intact, painful on percussion. The I degree of tooth mobili- Krok 2 Stomatology (англомовний варiант, iноземнi студенти) 2017 рiк 19 ty is observed; pus is being discharged from one month ago. In the morning the under the marginal gingiva of the 64 tooth. crunching is more frequent and decreases The alveolar process is deformed at its vesti- towards the evening. Objectively: the face bular and palatine surfaces. Make the provi- is symmetrical, the skin above the joint is sional diagnosis: unaltered, the mouth opens by 2,9 mm. What is the most likely diagnosis in this A. Acute odontogenic osteomyelitis case? B. Acute albuminous periostitis C. Acute suppurative periostitis A. Arthrosis D. Ossification periostitis B. Acute arthritis E. Ewing’s sarcoma C. Temporomandibular joint dislocation D. Chronic arthritis 140. A 4-year-old child has developed E. Pain dysfunction syndrome of the acute spontaneous pain in the tooth on the temporomandibular joint lower right jaw, which aggravates on biti- ng. Objectively: in the 85 tooth there is a 144. A 34-year-old man came to a dental deep carious cavity non-penetrating to the clinic for extraction of the 26 tooth. After dental cavity. Probing is sharply painful at application of 1,7 ml of Ultracain (Articai- all points of the cavity floor. Painful reacti- ne) solution for local anaesthesia the pati- on to cold water stimulus and percussion ent developed general fatigue and nausea. is observed; mucosa surrounding the 85 is Objectively: the skin is pale, cold, cyanotic, hyperemic. Submandibular lymphadenitis is covered in clammy sweat; BP is 60/40 mm detected. Make the provisional diagnosis: Hg. What urgent condition did the patient develop? A. Acute pulpitis complicated with peri- odontitis A. Collapse B. Acute albuminous periostitis B. Anaphylactic shock C. Acute serous periodontitis C. Loss of consciousness D. Acute suppurative pulpitis D. Bronchial asthma E. Exacerbation of chronic periodontitis E. Urticaria 141. An 18-year-old girl came to a dentist to 145. Carious cavities of the 11 and 21 teeth check the quality of fissure sealing that had were detected during the preventive exami- been performed one year ago. Objectively: nation of a 20-year-old patient. What materi- the sealant is completely retained in the 17, al should be used to fill the detected caviti- 16, 26, 27,37,and 47.No sealant was detected es? in the 36 and 46, in the distal longitudi- nal fissures of these teeth there is softened A. Microhybrid composite enamel. Fedorov-Volodkina Hygiene Index B. Macrofilled composite is 2,5. What tactics regarding the 36 and 46 C. Amalgam should the dentist choose in this case? D. Phosphate cement E. Plastic A. Preventive filling B. Repeated noninvasive sealing 146. A 14-year-old boy complains of rapid C. Invasive sealing wearing-off of tooth crowns. Objectively: D. Applications with fluorine-containing gel tooth crowns are worn-off by 1/3. Enamel E. Electrophoresis of calcium-containing easily chips off and is pale gray in color. solution Make the diagnosis: 142. A patient needs the 36 tooth extracted. A. Stainton-Capdepont syndrome After administering anaesthesia the doctor B. Dentinogenesis imperfecta started applying the elevator. However, C. Fluorosis immediately after that the patient suddenly D. Systemic hypoplasia paled, complained of dizziness, ear noise, E. Focal hypoplasia and blackout and slid down in the chair. What is the most likely diagnosis? 147. A patient complains of periodical gi- ngival hemorrhages during tooth brushing A. Unconsciousness and increased teeth sensitivity to thermal B. Anaphylactic shock and chemical stimuli, which persist for the C. Collapse last 6 years. On examination the gums are D. Shock swollen and hyperemic. Periodontal pockets E. Hypoglycemic coma are 5 mm deep with serous purulent content, tooth cervices are bared, I degree tooth 143. A 54-year-old patient complains of mobility is observed. On X-ray: irregular frequent crunching sound in the right resorption of of alveolar septa up to their temporomandibular joint, which developed 1/2. What diagnosis corresponds with the gi- Krok 2 Stomatology (англомовний варiант, iноземнi студенти) 2017 рiк 20 ven clinical presentation? A. Acute odontogenic maxillary sinusitis A. Exacerbation of generalized periodontitis, B. Acute rhinogenic maxillary sinusitis II degree C. Chronic odontogenic maxillary sinusitis B. Papillon-Lefevre syndrome D. Cyst of the maxillary sinus C. Periodontosis, I degree E. Acute ethmoiditis D. Exacerbation of generalized periodontitis, I degree 152. During application of tuberal E. Exacerbation of severe catarrhal gingivitis anaesthesia the patient developed rapidly increasing tissue edema and reduced mouth 148. A 63-year-old man complains of pain opening. What resulted in such a condition? in the area of maxillary mucogingival fold caused by using a removable lami- A. Vascular trauma nar denture. Objectively: in the area of the B. Muscle trauma during anaesthesia appli- mucogingival fold there is a trophic ulcer wi- cation th swollen margins and hemorrhaging floor. C. Nerve trunk trauma Make the diagnosis: D. Intolerance to the anaesthetic E. Anaphylactic shock A. Denture-related stomatitis B. Toxic chemical stomatitis 153. During application of infraorbital C. Toxic infectious stomatitis anaesthesia the patient developed a posti- D. Allergic contact stomatitis njection hematoma. What vessel had been E. Greenhouse effect damaged? 149. A 64-year-old man complains of pain A. Infraorbital artery and mobility of his front lower teeth. B. Maxillary artery Objectively: the 43, 42, 41, and 31 teeth C. Pterygoid venous plexus demonstrate the III degree mobility. These D. Temporal artery teeth are planned to be extracted and E. Palatine artery immediate-insertion denture is to be made. When should this denture be put in place? 154. A patient came to a dental surgeon with complaint of periodical pains in the 22 tooth. A. On the day of the teeth extraction On X-ray examination the patient was di- B. In 2-4 days after the teeth extraction agnosed with granulomatous periodontitis C. In 6-8 days after the teeth extraction developed due to filling material penetrati- D. In 3-6 days after the teeth extraction ng the space behind the root apex. What E. In 1-2 days after the teeth extraction further treatment tactics should be chosen? 150. Duringtheoralcavitysanationonthe A. Resection of the root apex vestibular surface of the 21 and 12 teeth in B. Extraction of the 22 tooth the cervical area there were detected chalky C. Prescription of anaesthetics spots. Enamel surface is dull and coarse, D. Referral to an oncologist no reaction to thermal stimuli. Decayed- E. Recurrent endodontic treatment Missing-Filled Index (dmft/DMFT) is 6, Hygiene Index is 2. Mesial occlusion is 155. After the inflammatory process in the observed. Within the first year of life the parotid area a woman developed frequent patient had been suffering from frequent pain attacks resembling electric current in casesofURTIandacaseofchicken her face on the right. The attacks last for 15- pox. What additional investigation methods 20 minutes. The most likely diagnosis is: would be useful for the diagnosis-making in A. Trigeminal neuralgia this case? B. Tympanic plexus neuralgia Trigeminal neuritis A. Vital staining C. B. Anamnesis data D. Exacerbation of chronic maxillary sinusi- Electric pulp test tis C. Exacerbation of chronic osteomyelitis D. X-ray E. E. Stomatoscopy 156. A 24-year-old woman came to a denti- 151. A patient complains of pain and sensati- st to receive sanation. Objectively on the on of heaviness in the left side of his face masticatory surface of the 37 tooth there and mucous discharge from the nose. On is a deep carious cavity connected with the examination: left cheek edema, destroyed dental cavity. The cavity probing is painless, 26 tooth. Tooth percussion is sharply painful. no reaction to thermal stimuli is observed X-ray demonstrates shadowed left maxillary in the tooth, percussion is painless. EOD is sinus. What disease corresponds with the gi- 108 microamperes. X-ray shows traces of fi- ven clinical presentation? lling material in the rooth canal of the 37 tooth, periodontal fissure is enlarged and Krok 2 Stomatology (англомовний варiант, iноземнi студенти) 2017 рiк 21 deformed. Make the diagnosis: of incisors. What preventive appliance would be optimal in this case? A. Chronic fibrous periodontitis of the 37 tooth A. Vestibulo-buccal shield B. Chronic granulating periodontitis of the B. Schonherr’s standard vestibular plate 37 tooth C. Frankel’s functional regulator C. Chronic granulomatous periodontitis of D. Janssen’s bionator the37tooth E. Rudolf’s plate with loops D. Chronic fibrous pulpitis of the 37 tooth E. Exacerbation of chronic granulomatous 161. A 45-year-old man complains of pain periodontitis of the 37 tooth and crepitation in the temporomandibular joint during the movements of the lower 157. A 40-year-old man came to an admi- jaw. Objectively: the face is symmetrical, the ssion room with an incised wound of the mouth opens with slight displacement to the infraorbital region received 8 hours ago. left. Dentition is intact. To clarify the di- On examination the wound underwent pri- agnosis X-ray of the temporomandibular joi- mary surgical treatment. In case of an inci- nt was performed. Where should the heads sed wound its edges: of the mandible be located normally during maximum mouth opening? A. Should not be excised B. Should be closed with secondary sutures A. At the top of the articular tubercle C. Should be closed with primary delayed B. In the center of the glenoid fossa sutures C. In front of the articular tubercle D. Should be processed with antibiotic D. In the center of the articular tubercle solution E. Closer to the distal part of the glenoid E. Should be cleaned and drained fossa 158. An ambulance has delivered an 8- 162. A 45-year-old man complains of pain year-old child to an admission room. An and crepitation in the temporomandibular oral surgeon has made the following di- joint during the movements of the lower agnosis: odontogenic phlegmon of the ri- jaw. Objectively: the face is symmetrical, ght submandibular area. What surgical the mouth opens with slight displacement approach would be advicable for open to the left. Dentition is intact. On occlusi- treatment of this phlegmon? ography there were detected centric and eccentric supracontacts. What treatment A. Dissection in the submandibular area, methods should be applied in the first place? parallel to the mandible B. Dissection parallel to the torus mandi- A. Selective teeth shaving bularis B. Mouthguard for muscle relaxation C. Dissection around the mandibular angle C. Appliances that limit mouth opening D. Dissection along the lower neck fold D. Mouthguards that increase the height of E. Dissection in the area of pterygomandi- central occlusion bular fold E. Lower jaw immobilization 159. A 14-year-old child complains of 163. A 65-year-old man during the throbbing undulating pain in the lower tooth extraction suddenly felt unwell, he left teeth, which aggravates due to hot developed severe pain irradiating to the stimuli. Objectively: on the masticatory left scapula and numbness of the left hand. surface of the 36 tooth there is a cari- Objectively: the patient is pale, beads of ous cavity within parapulpar dentin, whi- perspiration appeared on his forehead, BP ch is non-communicating with the dental is 170/90 mm Hg, heart rate is 86/min., cavity. The cavity floor probing is painless, rhythmical. The dentist stopped the mani- tooth percussion is painful. What treatment pulations in the oral cavity. What drug method would be optimal in the given case? should be administered in this case? A. Vital extirpation A. Nitroglycerine B. Devital extirpation B. Zelenin drops C. Devital amputation C. Valerian tincture D. Vital amputation D. Valocordin E. Biological method E. Analgin (Metamizole) 160. An orthodontist has registered for 164. A 45-year-old patient after admini- regular check-ups a 3,5-year-old child, stration of local anaesthesia in preparati- who has a bad habit of finger sucking on to oral surgery has suddenly felt unwell, and presents with infantile swallowing. On developed increasing edema of laryngeal examination: milk occlusion, direct contact mucosa and respiration disorder. The dentist Krok 2 Stomatology (англомовний варiант, iноземнi студенти) 2017 рiк 22 stopped the manipulations in the oral cavi- man. At the stage of placing the artificial ty. What type of asphyxia developed in the teeth in the dental articulator it is necessary patient? to determine sagittal articular angle. This angle equals: A. Stenotic B. Dislocational A. 20-40 C. Valvu lar B. 5-15 D. Obturative C. 15-20 E. Aspiration D. 20-25 E. 40-50 165. A 19-year-old young man complains of a fistula on the neck anterior surface, whi- 169. In a new neighbourhood unit of a large ch periodically reappears at the same place. city a dental clinic is being opened. The Objectively: at the neck midline between clinic will employ 3 prosthodontists. How the hyoid bone and thyroid cartilage there many positions of dental technicians, dental is a fistula; the skin of the affected area is nurses, and orderlies should be provided? scarred, drawn-in, and macerated. In the surrounding tissues a dense band extending A. 6 dental technicians, 1 dental nurse, 1 from the fistula opening to hyoid bone can orderly be palpated. A doctor has made a provisi- B. 3 dental technicians, 1 dental nurse, 1 onal diagnosis of thyroglossal fistula. Specify orderly the additional method of investigation: C. 3 dental technicians, 1,5 position of a dental nurse, 1 orderly A. Contrast radiography D. 6 dental technicians, 1 dental nurse, 0,5 B. Probing position of an orderly C. Computer tomography E. 1,5 position of a dental technician, dental D. Ultrasound nurse, and an orderly E. - 170. A 25-year-old man complains of short- 166. Mother of an 8-month-old girl came to term pain in the tooth on the lower ri- a clinic with complaints of the child’s anxi- ght jaw during eating sweet, hot, and cold ety, fussiness, high fever up to 38, 5oC,si- food. Objectively: in the 36 tooth on the gns of alimentary canal irritation, vomiti- distal surface there is a carious cavity ng and refusal to eat. On objective exami- non-communicating with the dental cavi- nation the child is pale, crying, presents wi- ty, dentin is softened. Probing of the cavi- th hyperemia, edema, gingival pain in the ty floor is painful, percussion is painless. frontal area of the upper jaw, no erupted Electric pulp test is 16 microamperes. Make teeth can be detected. Make the diagnosis: the final diagnosis: A. Hindered tooth eruption A. Acute deep caries B. Acute herpetic stomatitis B. Acute median caries C. Food poisoning C. Hyperemia of the pulp D. Hematogenous osteomyelitis of the maxi- D. Chronic gangrenous pulpitis lla E. Chronic fibrous periodontitis E. Odontogenic osteomyelitis of the maxilla 171. A 30-year-old woman complains of a 167. A 45-year-old patient complains of carious cavity in the 16 tooth, food retention inability to properly masticate due to the in the gap between the 16 and 17 teeth. Duri- loss of lateral teeth. The 17, 16, 15, 25, 26, ng examination there was detected a cari- 27, 37, 36, 35, 44, 45, and 46 teeth are mi- ous cavity within mantle dentin with wide ssing. The retained teeth exhibit the I-II opening on the approximal-medial surface degree of mobility. The patient is diagnosed of the 16 tooth. The cavity floor and walls with generalized periodontitis. Kennedy are pigmented, dense, painless on probing. class I dentition defects are observed. What Percussion of the 16 tooth is painless. On construction would be optimal in the given thermodiagnostics a short-term reaction can case? be observed. Make the diagnosis: A. Clasp-retained (bugel) removable partial A. Chronic deep caries denture with splinting elements B. Chronic fibrous pulpitis B. Partial laminar denture C. Chronic median caries C. Elbrecht’s dental splint D. Chronic fibrous periodontitis D. Mamlok’s dental splint E. Chronic concrementous pulpitis E. Cantilever dental bridge 172. A 40-year-old patient requires surgical 168. Removable complete laminar denture sanation of the oral cavity. Objectively: the is being made for a 63-year-old edentulous 36 tooth is completely destroyed. Mouth can Krok 2 Stomatology (англомовний варiант, iноземнi студенти) 2017 рiк 23 be fully opened. What anaesthesia would be superciliary area. General condition of the optimal for extraction of the 36 tooth? child is unaffected. Objectively: swelling of the forehead tissues spreading towards the A. Torusal left eyelids; the swelling is soft, fluctuati- B. Mandibular on sign is present. Make the preliminary di- C. Mental agnosis: D. Infiltration E. Berchet-Dubov A. Hematoma of the left superciliary area B. Hemangioma of the right superciliary area 173. A 45-year-old woman needs a denture. C. Fracture of the temporal bone Objectively: the 17, 16, 15, 14, 12, 25, and D. Fracture of the frontal bone 26 teeth are missing. Specify the Kennedy’s E. Hematic abscess of the left superciliary class of dentition defects in the given case: area A. II class, 2 subclass 178. Mother of a 10-year-old girl complai- B. II class, 4 subclass ns of a cosmetic defect of the child’s 22 C. III class, 1 subclass tooth that erupted with damaged enamel. D. III class, 3 subclass Anamnesis states premature extraction of E. II class, 3 subclass the 62 tooth due to caries complication. There is a white-yellow spot with clear 174. A 37-year-old patient complains of an margins on the vestibular surface of the 22 aesthetic defect. Objectively: the 13 tooth is tooth. Enamel retains glossiness, no surface destroyed by 2/3. The tooth is pulpless, the roughness can be detected on probing. Make root canal is filled. How deep should the root the diagnosis: canal be opened for pivot crown installation in this patient? A. Local enamel hypoplasia B. Fluorosis A. 2/3 of the root canal C. Acute superficial caries B. 1/3 of the root canal D. Chronic superficial caries C. 3/4 of the root canal E. Systemic enamel hypoplasia D. 1/2 of the root canal E. Full length of the root canal 179. A 48-year-old patient has come to a hospital with complaints of defects in the 175. A 10-year-old boy complains of acute paragingival area and slight sensitivity to pain attacks in the area of his upper left thermal stimuli. Objectively: there are hard teeth. The toothache persisted for a night. tissue defects that resemble a wedge with Objective examination revealed a carious smooth polished walls on the precervical cavity on the masticatory surface of the 26 vestibular surface of the 23 and 24 teeth. tooth within parapulpar dentin. Probing is Thermal test is slightly positive. What is the sharply painful at all points of the cavity most likely diagnosis? floor. Markedly positive reaction to cold water stimulus is observed. Select the most A. Cuneiform defect likely diagnosis: B. Enamel necrosis C. Acute deep caries A. Acute diffuse pulpitis D. Enamel erosion B. Acute serous periodontitis E. Endemic fluorosis C. Acute suppurative pulpitis D. Acute suppurative periodontitis 180. Parents of a 3-year-old child complain E. Acute local pulpitis that the child has a neck growth that developed 3 months after the birth. Objecti- 176. A 15-year-old patient complains of cari- vely: in the upper lateral neck area there is a ous cavity and short-term ”lightning-fast” semicircular neoplasm with limited mobility, pain attacks in the 26 tooth. The pain attacks soft elastic consistency, no skin alterations, cease in 1-2 minutes after eating. Objecti- painless on palpation. Puncture yielded pus- vely: there is a deep carious cavity filled with like clear yellow substance. Make the provi- softened dentin. The cavity floor is painful sional diagnosis: on probing. Make the diagnosis: A. Branchial cleft cyst A. Pulpal hyperemia B. Chronic lymphadenitis B. Acute traumatic pulpitis C. Lymphangioma C. Acute suppurative pulpitis D. Specific lymphadenitis D. Acute local pulpitis E. Hemangioma E. Acute diffuse pulpitis 181. A 16-year-old adolescent girl complai- 177. A 7-year-old girl hit her forehead one ns of pain caused by cold stimuli and food day ago. Several hours after the sustained particles retained in her upper jaw tooth. trauma a swelling developed in the left Krok 2 Stomatology (англомовний варiант, iноземнi студенти) 2017 рiк 24

Objectively: on the contact surface of the 185. During preventive examination a 24 tooth there is a carious cavity within patient was diagnosed with precancerous parapulpar dentin. The cavity floor and walls hyperkeratosis of the red border of the are covered with light softened dentin. The lower lip. What treatment should be prescri- carious cavity floor is sensitive to probing, bed? percussion of the 24 is painless. Cold water stimulus is painful, the pain quickly abates A. Surgical removal of the focus within after the stimulus is removed. Make the di- healthy tissues agnosis: B. Surgical removal of the focus wi- thin healthy tissues + close-focus A. Acute deep caries roentgenotherapy B. Acute median caries C. No treatment is required C. Acute diffuse pulpitis D. Surgical removal of the focus within D. Chronic fibrous pulpitis healthy tissues + chemotherapy E. Chronic deep caries E. Palliative treatment 182. A 15-year-old girl complains of brief 186. A 23-year-old patient is hospitalized pain attacks in her teeth due to chemical into a dentofacial unit with provisional di- stimuli. Objectively: on the contact surfaces agnosis of the II degree thermal burns of the of the 11, 21, and 22 teeth there are enamel right buccal and parotid-masseter region. areas matt white in color, with lost shine, What scar tissue will develop in this case? covered in large amount of dental deposit. Enamel is softened and can be easily chi- A. Healing without a scar pped off with excavator. Probing of lesions B. Atrophic scar is painless. Percussion is painless. No reacti- C. Hypertrophic scar on to cold stimuli. Make the diagnosis: D. Hypotrophic scar E. Keloid scar A. Acute superficial caries B. Acute median caries 187. A 56-year-old man complains of pain C. Acute initial caries in the gonial angle. Objectively: the 45, 46, D. Chronic initial caries 34, 35, and 36 teeth are missing; there are E. Chronic superficial caries slight swelling and reduced mouth opening observed. X-ray: right-sided fracture of the 183. A 32-year-old patient addressed a body of mandible in the area of the 45 and dentist with complaints of inability to close 46 teeth without bone defect. What dental his mouth. Objectively the mouth is half- splint should be used for treatment? open, the chin is protruding forwards and is displaced to the left. Such condition A. Weber occurred after the mouth was opened wi- B. Limberg de. What is the most likely diagnosis? C. Tigerstedt D. Zbarzh A. Anterior right-sided mandibular dislocati- E. Vankevych on B. Anterior left-sided mandibular dislocation 188. A clasp-retained (bugel) removable C. Anterior bilateral mandibular dislocation partial denture for the lower jaw is to be D. Posterior right-sided mandibular dislocati- made for a 53-year-old patient. Objectively: on the 38, 37,35, 34, 45, 46, and 47 teeth are mi- E. Posterior left-sided mandibular dislocation ssing. The retained teeth are firm, with low clinical crowns. What fixation method of the 184. A 37-year-old patient has symmetrical denture would be optimal in this case? face; the mucosa in the area of the 12 tooth root apex projection is pale pink; palpation A. Telescopic system is painless; the tooth crown is destroyed by B. Supporting-retaining clasps (clammers) 1/3; percussion is painless. X-ray: the root C. Bar system canal of the 12 tooth is filled to the apex; D. Attachments granuloma 4 mm in diameter surrounds the E. Ball joint attachments root apex. Choose the method of surgical treatment: 189. A 28-year-old woman is diagnosed with chronic generalized periodontitis, II degree. A. Granuloma removal with root apex The doctor prescribed her a mouthwash with resection chlorhexidine gluconate as a part of complex B. Root hemisection therapy. This drug belongs to the following C. Coronary radicular tooth separation group of antiseptics: D. Root amputation E. Tooth extraction Krok 2 Stomatology (англомовний варiант, iноземнi студенти) 2017 рiк 25

A. Detergents B. Dyes A. II C. Halogens B. I D. Oxidants C. III A E. Acids and alkalis D. III B E. IV 190. A 22-year-old patient has suffered uni- lateral linear fracture in the area of the 194. A 19-year-old patient came to a gonial angle. Immobilization was provi- dentofacial clinic with complaints of pain ded with full dental brace with loops and in the gonial angle on the right, impaired intermaxillary elastic expansion. Recovery mouth opening and painful chewing. The si- was uncomplicated. The brace should be gns had been persisting for 5 days, emerged removed after: spontaneously and had been aggravating gradually. Mandibular contracture is of the A. 3 weeks III degree. On examination of the oral cavi- B. 2 weeks ty: hyperemia, edema of the retromolar C. 1 week space on the right, hood-shaped mucosa D. 10 days from under which pus is being discharged E. - and 2 tooth tubercles can be detected. X-ray shows oblique medial tooth position. Make 191. A 33-year-old man, a metalworker, the diagnosis: complains of pain and itching in the gums, gingival hemorrhages intensifying during A. Acute suppurative of the 48 tooth brushing. The onset of the disease tooth was 1 year ago. Objectively: the gums in the B. Acute suppurative periostitis from the 48 area of upper and lower frontal teeth are tooth hyperemic, swollen, and cyanotic. There are C. Chronic local mandibular osteomyelitis significant mineralized deposits on the teeth; D. Fracture of the gonial angle the periodontal sockets are 3 mm deep and E. Acute submandibular sialadenitis produce small amount of serous discharge. What is the most likely diagnosis? 195. A 45-year-old man came to a dentist with complaints of a massive and extremely A. Chronic generalized periodontitis, I class dense (resembling wood) infiltration in the B. Chronic localized periodontitis, I class parotid and retromandibular areas, which C. Exacerbation of chronic generalized persists for 1,5 months. The patient’s general periodontitis, II class condition remains largely undisturbed, signs D. Acute localized periodontitis, II class of inflammatory process are vague and indi- E. Generalized periodontosis, I class stinct. Periodically in the infiltration area the skin assumes cyanotic-purple color, a 192. A 34-year-old man complains of pain in soft patch appears in the center, where 1-2 the area of his right eye, headache, and body o fistulae develop and discharge pus with whi- temperature rise up to 38,6 C. Two days ago te granules. Periodically fistulae close and the patient developed an infiltration in the reopen. Make the diagnosis: lower eyelid of the right eye. Objectively the eyelids are markedly swollen, palpebral A. Parotid actinomycosis fissure is closed, conjunctiva is swollen. B. Parotid tuberculosis Exophthalmos is observed. The eyeball is C. Parotid erysipelas immobile, vision is impaired. Make the di- D. Chronic parotid sialadenitis agnosis: E. Adenocarcinoma of the parotid gland A. Orbital phlegmon 196. Parents of a 6-year-old child complain B. Eyelid phlegmon of their child having a gradually enlargi- C. Purulent maxillary sinusitis ng neoplasm in the left parotid-masticatory D. Angular vein trombophlebitis region. Skin over the tumor is without di- E. Lower eyelid abscess scoloration. The tumor is painless, but when the head bends down the tumor increases in 193. A 35-year-old patient complains of size and assumes bluish coloring. What di- burns of the face and neck, swelling and sease can be suspected in the child? burning pain in the affected area. On examination: edema of the face and neck, A. Hemangioma palpebral fissure is narrowed due to swelli- B. Fibroma ng, affected skin is hyperemic and covered C. Atheroma with strained thin-walled blisters filled wi- D. Lymphangioma th clear content. Where blisters are broken, E. Cyst of the parotid gland there are pink wounds, sharply painful to touch. Determine the degree of the burns: 197. A 40-year-old patient complains of pain Krok 2 Stomatology (англомовний варiант, iноземнi студенти) 2017 рiк 26 in the tragus area, clicking sound during tonsils, and posterior wall of the pharynx. mouth opening, stuffed ears. Objectively: Submandibular, submental and deep cervi- the face is symmetrical, mouth opening path cal lymph nodes have been enlarged for is straight. Dentition defect can be estimated 4 months. Two weeks ago the patient as Kennedy I class; the 18, 17, 16, 26, 27, developed intermittent fever and general and 28 teeth are missing. In this case the fatigue. Select the correct sequence of HIV load would be the most traumatizing for the diagnosing: following anatomical structure: A. Enzymoimmunoassay, immunoblotting A. Interarticular disk (Western-Blot) B. Articular capsule B. CD4 cell count, enzymoimmunoassay C. Articular head C. Complete blood count, viral load D. Distal slope of the articular tubercle D. Complete blood count, enzymoi- E. Socket floor of the temporal bone mmunoassay E. Viral cultivation, enzymoimmunoassay 198. A 50-year-old man complains of bared dental cervices on his upper and lower 200. A 38-year-old man complains of jaws. Objectively: the teeth and dentition sensation of a foreign body on his tongue are intact, clinical crowns are elongated, and development of gag reflex during talki- the teeth have no pathologic mobility, are ng. The signs appeared after the prolonged worn off within the physiological norm. taking of antibiotics. Objective examinati- To remove supracontacts it is planned to on detected thickened and pigmented fi- perform selective teeth shaving. What addi- liform papillae enlarged to 2-3 cm in si- tional investigation is necessary in the given ze. Histologically papillar hyperplasia and case? marked keratinization without alteration of the surrounding tissues were detected. What A. Occlusiography is the most likely diagnosis? B. X-ray C. Masticatiography A. D. Gnathodynamometry B. Median rhomboid E. Mastication tests C. D. Glossitis areata exfoliativa 199. A 28-year-old man presents with E. Geographic tongue profuse caseous coating on the posterior third of the back of his tongue, soft palate, INSTRUCTIONAL BOOK

Ministry of public health of Ukraine (MPH of Ukraine) Department of human recources policy, education and science Testing Board

TEST ITEMS FOR LICENSING EXAMINATION: KROK 2. STOMATOLOGY.

Kyiv. Testing Board. (English language).

Approved to print 03.04./№47. Paper size 60х84 1/8 Offset paper. Typeface. Times New Roman Cyr. Offset print. Conditional print pages 24. Accounting publishing pages 28. Issue. 297 copies. List of abbreviations

A/G Albumin/globulin ratio HR Heart rate A-ANON Alcoholics anonymous IDDM Insulin dependent diabetes mellitus ACT Abdominal computed tomography IFA Immunofluorescence assay ADP Adenosine diphosphate IHD Ischemic heart disease ALT Alanin aminotranspherase IU International unit AMP Adenosine monophosphate LDH Lactate dehydrogenase AP Action potential MSEC Medical and sanitary expert committee ARF Acute renal failure NAD Nicotine amide adenine dinucleotide AST Aspartat aminotranspherase NADPH Nicotine amide adenine dinucleotide phosphate restored ATP Adenosine triphosphate NIDDM Non-Insulin dependent diabetes mellitus BP Blood pressure PAC Polyunsaturated aromatic carbohydrates bpm Beats per minute PAS Periodic acid & shiff reaction

C.I. Color Index pCO2 CO2 partial pressure

CBC Complete blood count pO2 CO2 partial pressure CHF Chronic heart failure pm Per minute CT Computer tomography Ps Pulse rate DIC Disseminated intravascular coagualtion r Roentgen DCC Doctoral controlling committee RBC Red blood count DM-2 Non-Insulin dependent diabetes mellitus RDHA Reverse direct hemagglutination assay DTP Anti diphtheria-tetanus vaccine Rh Rhesus ECG Electrocardiogram (R)CFT Reiter's complement fixation test ESR Erythrocyte sedimentation rate RIHA Reverse indirect hemagglutination assay FC Function class RNA Ribonucleic acid FAD Flavin adenine dinucleotide RR Respiratory rate

FADH2 Flavin adenine dinucleotide restored S1 Heart sound 1 FEGDS Fibro-esphago-gastro-duodenoscopy S2 Heart sound 2

FMNH2 Flavin mononucleotide restored TU Tuberculin unit GIT U Unit Gy Gray USI Ultrasound investigation GMP Guanosine monophosphate V/f Vision field Hb Hemoglobin WBC White blood count HbA1c Glycosylated hemoglobin X-ray Roentgenogram Hct Hematocrit HIV Human immunodeficiency virus